Наука


Ответить в тред Ответить в тред

<<
Назад | Вниз | Каталог | Обновить тред | Автообновление
521 54 175

Тред тупых вопрос Прошлый https://2ch.hk/sci/res/456196.html Аноним 31/01/19 Чтв 11:54:31 4587001
15488683351890.jpg (43Кб, 720x654)
720x654
Аноним 31/01/19 Чтв 12:07:08 4587022
Так, фотон квант электромагнитного поля, электрон-квант электрического поля. Но есть и магнитное поле(или нет?), у него есть квант?
И если есть электромагнитное поле то почему выделяют отдельно электрическое и магнитное? Или если есть электрическое и магнитное, то зачем их объединяют в электромагнитное?
Аноним 31/01/19 Чтв 12:25:07 4587043
benny.png (78Кб, 618x351)
618x351
>>458700 (OP)
Почему среди Нобелевских лауреатов по естественным наукам 25% - евреи?
Несколько более высокий средний IQ ничего не объясняет - они составляют ничтожную долю населения развитых стран, людей с высоким IQ там во много раз больше чем евреев вообще.
"Культура" и "воспитание" это не ответ non-answer.
Каково же научное объяснение?
Аноним 31/01/19 Чтв 12:30:38 4587054
>>458704
Они не работают.
А учитывая высокий айкью-чем ещё заниматься?
Или наукой, или только всякие захваты власти и перевороты с геноцидами и порабощениями хитростью и многоходовочками устраивать.
В среднем.
Аноним 31/01/19 Чтв 12:34:23 4587065
>>458705
1. Евреи настолько мощны, что им не надо работать, вообще, одно только еврейство неплохо кормит.
2. Для евреев теоретическая физика - не работа, а так, разминка для ума.
3. От нечего делать, походя, евреи двигают науку семимильными шагами.
Г*споди, вот это сверхлюди! Какое счастье, что они над нами властвуют.
Аноним 31/01/19 Чтв 13:08:58 4587076
>>458704
Развитое еврейское лобби в передовых странах. Гоям как бы похуй кто там будет считаться крутым, поэтому они особо не парятся, жиды же всеми силами продвигают своих, поэтому получается так.
Аноним 31/01/19 Чтв 16:08:38 4587287
>>458702
Электрон - квант элктронного поля, ну или лептонного. Вопрос закрыт.
Аноним 31/01/19 Чтв 16:18:15 4587298
>>458728
>Вопрос закрыт.
Этот да. Но там кроме него много других вопросов.
Аноним 31/01/19 Чтв 18:04:32 4587539
>>458729
>>458729
>Этот да. Но там кроме него много других вопросов.
>электрон-квант электрического поля. Но есть и магнитное поле(или нет?), у него есть квант?
У магнитного и у электрического поля есть квант, называется фотон. Электрическое и Магнитное поле это 2 ортогональные компоненты одного единого электромагнитного поля.
Через электромагнитное поле взаимодействуют заряженные частицы, такие как электрон.
Заряд не создает вокруг себя электромагнитное поле, поэтому электрон не может являться частью электрического или магнитного поля. Грубо говоря, заряд взаимодейсвтует с полем, возбуждет его и создает энергетический потенциал вокруг, другие заряженные частицы могут чувствовать этот потенциал. Посредством электромагнитного поля, квантом которого является фотон, такие частицы могут взаимодействоать.
не знаю насколько просто и могу ли еще проще объяснить.
Аноним 31/01/19 Чтв 18:09:13 45875410
>>458702
>И если есть электромагнитное поле то почему выделяют отдельно электрическое и магнитное? Или если есть электрическое и магнитное, то зачем их объединяют в электромагнитное?
Потому что в четырехмерном виде, это поле прекрасно выглядит как обычное векторное поле ( функция от 4 координат, выдает 4 значения). А вот когда мы уже переходим к мир земной, то формально отделяют кусок временной (электрический потенциал, одно измерение, это скалярное поле), а три других отправляются в магнитную составляющую ( векторное поле). Делят их, так как одна часть действует силой зависящей только от заряда, а вторая часть, еще и от скорости заряда зависит.
Аноним 31/01/19 Чтв 18:58:56 45876211
>>458754
дополню.
В 4х мерном виде это не вектор, а тензор, записываемый через 4 вектор потенциал, где 0 компорнента скаляр через которые, да, определяется электрическое поле, а 1,2,3 вектор, через которые выделяется магнитное. Оба элемента электромагнитного поля являются векторами, т.е напряженность и магнитная индукция. И напряженность и магнитная индукция создают векторные поля, которые являются ортогональными.
Аноним 31/01/19 Чтв 22:03:18 45877312
>>458762
Ноуп. ЭМ поле это все таки 4-вектор. Но он сам по себе не интересен, а его проявление описывается только его дифференциальная форма, которая как раз и является тензором. И в силу (анти)симметрии тензора ЭМ и его линейности 4-вектор определяется неоднозначно и он нем вообще забывают.
Аноним 31/01/19 Чтв 22:59:43 45878213
>>458773
Ну да, просто меня смутило, что ты говоришь о потенциале, упомянул электрический потенциал и назвал его скалярным полем. Даже если выражается он через скаляр, то напряженность электромагнитного поля это как раз таки векторное поле. там градиент скаляра.
В классическом понимнии электромагнитное поле это как раз напряженность по одной оси и индукция по другой. И человек, который не решал задачки, а просто имеет стандартное понимания электромагнитного поля или даже просто излучения, в теории поля может смутиться такими словами мол поле скалярное. Оба этих поля электрические и магнитное является векторными, но origin конечно у них разный, если сомтреть на потенциал.
Аноним 01/02/19 Птн 00:40:38 45879614
>>458782
Я другой анон, лол.
Вообще поле в строгом физическом понимании это некоторая функция от координат, а если уходить в дебри теормеха, это некоторое гладкое н-мерное множество. Потенциал же это просто точка на "поверхности" этого множества.
Всякие вектора напряженности это уже следствие некоторых преобразований, связанные с выбором системы отсчета, относительно этой точки. Полем в строгом смысле они не являются, хоть и являются некоторым его проявлением.
Аноним 01/02/19 Птн 02:26:45 45880615
15468864387980.png (532Кб, 700x879)
700x879
>>458753
>У магнитного и у электрического поля есть квант, называется фотон. Электрическое и Магнитное поле это 2 ортогональные компоненты одного единого электромагнитного поля.
Получается, магнит магнитит фотонами?...
Аноним 01/02/19 Птн 02:49:04 45880816
>>458806
Лол ну ты так это сформулировал, ответ может показаться смешным. Но да, фотонами, если откинуть в сторону всякие уточнения и слова типо "виртуальный".
Аноним 01/02/19 Птн 04:36:00 45881217
Кто или что прикладывает силу сильного ядерного взаимодействия к протона, например?
Аноним 01/02/19 Птн 04:36:30 45881318
>>458812
>к протонам
Быстрофикс
Аноним 01/02/19 Птн 07:47:30 45881819
>>458808
>Но да, фотонами
И какая у них длина волны? Почему они движутся не прямолинейно? Почему не взаимолействуют с немагнитной материей?

Зачем называть фотонами то, что не имеет никаких их свойств кроме распространения со скоростью 300к км/сек?
Аноним 01/02/19 Птн 08:07:06 45881920
Если это существует/происходит, то это можно описать именно так, как оно реально происходит, а не абстрактно.
Да/нет?
Аноним 01/02/19 Птн 12:07:17 45883921
>>458818
>Зачем называть фотонами то, что не имеет никаких их свойств кроме распространения со скоростью 300к км/сек?
Нет, фотонами называется все что имеент 1h спин, нулевую массу и нулевой заряд.
>Зачем называть фотонами
Такая вот квантовая теория поля
>И какая у них длина волны?
Это понятие классической физики. Если ты возьмешь комптоновкую длину волны, то будет бесконечность, из чего типо следует неограниченный радиус взаимодейсвтия, но это карчое глупости, т.к. понятие длины волны это чисто классическая физики.
>Почему они движутся не прямолинейно?
Они вообще виртуальные.
>Почему не взаимолействуют с немагнитной материей?
Почему не взаимодействуют? Еще как взаимодейсвтует. Просто немагнитная материя, это я понимаю ферромагнетики в твоем понимании, это надо лезть совсем в другие дебри, чтобы понять почему что-то магнитит, а что-то нет.
Аноним 01/02/19 Птн 12:10:28 45884022
>>458839
>почему что-то магнитит, а что-то нет
Потому что СТО если совсем до корней рыть
Аноним 01/02/19 Птн 12:11:08 45884123
>>458818
>Почему они движутся не прямолинейно?
а ну если ты будешь смотреть вообще движение фотонов, типо свет, то всегда прямолинейно нулевая геодезическая
Аноним 01/02/19 Птн 13:01:24 45884624
>>458812
>сильного ядерного взаимодействия
Сильное ядерное взаимодейсвтие.
Аноним 01/02/19 Птн 13:04:30 45884725
>>458841
>а ну если ты будешь смотреть вообще движение фотонов, типо свет, то всегда прямолинейно нулевая геодезическая
Я про "магнитные фотоны"
Аноним 01/02/19 Птн 13:36:53 45885026
>>458847
Нет магнитных немагнитных, они просто фотоны. И
>Они вообще виртуальные.
И я, наверное, начинаю понимать что ты вдруг представил под фотонами - силовые линии. Это не то. Это как раз модификация поля, как говорили аноны выше.
Аноним 01/02/19 Птн 14:22:17 45885527
>>458846
> Сильное ядерное взаимодейсвтие.
Это название силы. А что прикладывает эту силу?
Аноним 01/02/19 Птн 14:41:24 45885728
>>458855
Преобразование поля :3
Аноним 01/02/19 Птн 18:04:25 45887129
А есть какие-нибудь годные лекции лжеученых? Причем не просто видео уровня "дядя стоит и рассказывает про память воды", а что-то ПОХОЖЕЕ на настоящую науку, чтобы лектор приводил какие-то аргументы, что-то доказывал. А на деле это была бы лютая хуита, от которой настоящий специалист в области охуеет.
Аноним 01/02/19 Птн 18:27:26 45887230
Я пацаны, я понимаю, что вопрос уровня /zog, но если ИИ так сильно нужен, почему нельзя было сделать git/svn + форум, чтобы толпы программистов вместе писали ИИ? Я думаю за 10 лет, при том кол-ве программистов, что мы имеем, при том кол-ве интересующихся ИИ, мы бы давно уже сделали несколько модулей ИИ. А так, никто из обычных людей даже не знает, что в ИИ еще не изобретено и очень все гордятся нейросетями. Я не говорю, что нейросети это плохо, я про то, что практика покажет, что еще не открыто, не изобретено и т.д. Такого чудесного поля для открытий, к сожалению, нет у других ученых, где можно экспериментировать не тратя на это годы, а просто можно направить толпу на написание одного модуля, провести один эксперимент, потом другой. Для этого надо сделать что-то типа списка лиц, которые ведут ИИ, как это сделано со свободными дистрибутивами. Я не претендую на то, что я умен, но типа для меня это очевидное решение проблемы ИИ. Или же кому-то неудобен вопрос ИИ? Просто государства, при всем их могуществе, давно бы потратили деньги, ресурсы СМИ и т.д., все, чтобы сделать такой грандиозный проект, который сделает шаг вперед для всей цивилизации. Может у государства есть ИИ, а у нас нет? Это как экономические форумы. Вы видели их в интернете? Вот это уже вопрос /zog, так что интернетом таки управляют.
Аноним 01/02/19 Птн 18:32:08 45887531
>>458871
Годных нет и не будет по определению.
Аноним 01/02/19 Птн 18:42:00 45887732
>>458872
Что ты понимаешь под ИИ?
Аноним 01/02/19 Птн 18:42:17 45887833
>>458871
Как-то раз видел охуенную лекцию. Там чувак пояснял, что у человека якобы нет инстинктов. Короче софизм за софизмом, а он якобы биолог и доказывает. Мудила ебаная. Так людей за нос водит. Любой психолог и биолог тебе дал бы ему пососать за такое
Аноним 01/02/19 Птн 18:48:06 45888034
>>458877
На самом деле ты задал очень хороший вопрос. Определю понятие ИИ так: программа, которая подобно человеку может читать книги, статьи, научные формулы и т.д., выдвигать гипотезы, выводить что-то дедуктивно, учиться инструкциям по выполнению чего-либо из естественного языка. Таким образом наш ИИ будет уметь в науку, будет уметь писать книги, программы (при достаточно хорошо написанном ТЗ и достаточном количестве уточняющих вопросов со стороны ИИ). Грубо говоря, википедия станет выдавать охуенные идеи. В понимании других людей ИИ - это программа, которая может вести разговор с человеком, но даже это никто не реализовал. Ты представляешь на сколько нас тормозят? А ведь организовать это просто. Есть куча лингвистов, программистов, психологов, которые быстро бы все разработали. В общем, тема превращается в /zog.
И как я говорил, вопрос в модульности. Мое определение понятия ИИ может идти рядом с определениями понятия ИИ других людей, в конечном итоге, полностью скопировать этими модулями и переплюнуть человека.
Аноним 01/02/19 Птн 18:54:45 45888335
>>458878
Ну вообще у человека нет инстинктов. Название поменяли. Инстинкт по-определению автоматическое действие наследуемое организмом. Когда как человек в силах его не совершать подключая высшую нервную систему. Для нас они проявляются в виде сильных желаний, но эти желания человек может нарушать, так что да. (Есть вроде парочка действий, которые больше рефлексы, короткие и некотролируемые, но это не сложные же действия)
Аноним 01/02/19 Птн 18:57:28 45888436
>>458880
>Определю понятие ИИ так: программа, которая подобно человеку может читать книги, статьи, научные формулы и т.д., выдвигать гипотезы, выводить что-то дедуктивно, учиться инструкциям по выполнению чего-либо из естественного языка.
Увы, это невозможно.
Аноним 01/02/19 Птн 19:04:43 45888537
>>458883
Это сейчас понятие инстинкт определяют так, чтобы человек не попадал под это понятие. А так - смех. Если соблюдаются определенные условия, человек ржет. Вместо инстинктов? У человека масса инстинктов. Все зависит от силы подаваемого стимула. Если в условиях депривации человеку продемонстрировать стимул, он будет реагировать также автоматически как человек. Если красивая женщина улыбнется мужику в условиях депривации, у того тоже будет улыбка. У котов же есть инстинкт играться, если кота наказать, инстинктивное поведение не будет демонстрироваться. Закономерности инстинктов у собак и животных одни и те же. Нет никакой разницы вообще. Псевдонаучные доводы о том, что инстинкт - это прямо полностью автоматическая реакция, которую невозможно прервать безосновательны. Любой инстинкт можно прервать в ходе его исполнения, именно потому человеческие инстинкты "не полностью автоматические". Человек может прервать исполнение инстинкта в любой момент, также как и животные. Опять же, многие инстинкты наказуемы.

>>458884
Это еще почему? Человеческий мозг - это просто огромное кол-во проводов и программ. В чем сложность сделать подобное? Даже если это сложно. Интеллект есть просто определение огромного количества понятий, которые сфокусированы в определенной структуре, которая функционирует в алгоритме. Интеллект необходимый для выдвижения гипотез это просто совокупность понятий и логических форм. Сложности для этого нет.
Читать же сейчас могут уже сканеры, которые сами определяют где какой рисунок. В чем сложность? Где доказательства того, что ИИ невозможен?
Аноним 01/02/19 Птн 19:09:48 45888738
Стикер (127Кб, 320x320)
320x320
>>458885
>Человеческий мозг - это просто огромное кол-во проводов и программ.
Ну приехали блять.
Аноним 01/02/19 Птн 19:15:15 45888839
>>458887
Да ладно бля? А что такое мозг по твоему? Мозг функционирует за счет электрических сигналов, которые где-то генерирует, где-то принимает. Это типа не так? Все твое сознание - это просто электрический потенциал в определенный момент времени. Прерви электрическую цепь, мозг перестанет функционировать так как прежде, если не умрет совсем.
Ты какой-то долбоеб для этого раздела. ИИ у него не возможен, мозг - не провода, не программы.
Аноним 01/02/19 Птн 19:19:52 45888940
>>458885
В этом посте, ты показал свое максимальное невежество в области биологии. Но дело даже не в этом. После этого обосрался в области физики. Собственно ты типичный представить поколения, верующего в всемогущего комплюктера, что вот однажды сделают великий ИИ, которые решит все твои проблемы. Нет не сделают. Причины в том, что если бы все тобою сказанное было правдой, то работа мозга уже бы сейчас была максимально подробно описана. Это конечно же не так. Что тут можно посоветовать. Ну пожалуй, прежде всего осознать в чем твои проблемы, и в чем кроется твой эскапизм и со всем этим идти к хорошему психотерапевту.
Аноним 01/02/19 Птн 19:25:09 45889141
>>458889
>В этом посте, ты показал свое максимальное невежество в области биологии
Обоснуй
>После этого обосрался в области физики.
каким образом?
>Собственно ты типичный представить поколения
Дело не в поколении
>верующего в всемогущего комплюктера
Чтобы такого нет, есть теория алгоритмов, которая показывает, что можно рассчитать компьютером, а что нет.
>если бы все тобою сказанное было правдой, то работа мозга уже бы сейчас была максимально подробно описана
Итак подробно описывают, но сейчас нельзя даже параллельно подключать вольтметр к мозгу или же измерить силу тока перебивая цепь где-нибудь в центре мозга. Ты какой-то тупой, раз не видишь очевидных проблем методологии.
>Ну пожалуй, прежде всего осознать в чем твои проблемы,
Ad homeniem! Обосрался? Обтекай, долбоеб. Нет аргумента - стяни ебало и думай башкой. Не можешь придумать - не разивай ебальника.
Аноним 01/02/19 Птн 19:28:24 45889242
Аноним 01/02/19 Птн 19:33:38 45889443
>>458892
ИИ копирует не химию. А химия в мозгу, в конечном счете, влияет или не влияет на сознание, которое уже является электрическим потенциалом или же влияет на запись данных. Зачем химию мозга имитировать? Компьютер в этом плане совершеннее.
Аноним 01/02/19 Птн 19:33:47 45889544
>>458891
У меня нет ни времени(5-6 лет), ни желания, чтобы обучать какого-то неуча основам биологии, физики и прочего. Это задача университетов. Но я не виню тебя, агрессивной невежество вполне обыденно здесь.
Аноним 01/02/19 Птн 19:37:26 45889745
>>458895
У тебя опять нет аргументов и ты срешь в штаны. У тебя вообще рефлексии нет? Опять аргументация к личности. Ты совсем ебанутый? Туда ли ты зашел? Как ты вообще про науку можешь хоть что-то пиздеть, долбоеб? Либо ты обосновываешь, либо ты стягиваешь ебало. И есть такая штука - бремя доказательства. Ты его на меня перекладываешь, мудак. С хуя я должен доказывать, что я тупой? Или какие-то университеты? Ты спизданул - ты доказывай. Вот научный метод, говна ты кусок.
Аноним 01/02/19 Птн 19:39:01 45889846
>>458894
>влияет или не влияет на сознание, которое уже является электрическим потенциалом
Ровно настолько же, насколько твоё очко является писькой, на самом-то деле.
Аноним 01/02/19 Птн 19:40:27 45889947
>>458897
Еще и буйный. Я бы с твоим неврозом таблеточки попил. И всё таки обратись к психотерапевту, чую у тебя ворох проблем в голове.
Аноним 01/02/19 Птн 19:40:33 45890048
>>458894
Ууу... пиздец... какой. Пишешь ты а стыдно мне.
Аноним 01/02/19 Птн 19:42:50 45890149
>>458900
>>458899
>>458898
Тред одного семена. Никакой аргументации, кроме ad homeniem.
Аноним 01/02/19 Птн 19:45:30 45890450
>>458901
Пойми одну простую вещь: ты пишешь бред. Это видно невооруженным глазом. Этим бредом ты показываешь, что не понимаешь ничего в биологии, ни в химии, ни в физике. Пассажи про электрический потенциал та еще смехота. Никто не будет тратить время. Хочешь разбираться с вопросами мозга, берешь учебники по биологии, биохимии и.т.д(либо поступаешь на какой-нибудь биофак/химфак). То что тебе кто-то забесплатно, на анонимном форуме для анимедвочек будет проводить лекции по основам тех или иных наук, надеяться опрометчиво.
Аноним 01/02/19 Птн 19:50:24 45890651
>>458904
Опять нихуя не обосновал. Ты за собой вообще не замечаешь, что ты несешь хуйню? Давай я тебе такую же хуйню напишу?
Внимание:

Ты русский язык вообще не знаешь. Эти знаки припенения, тебе любой школьник скажет, что так только долбоеб делает. Тебя на смех любой сосницкий поднимет. Иди бери учебник Розенталя и больше сюда не возвращайся.

Это форма твоей "научной" аргументации. Пацаны, простите пожалуйста за то, что трачу тред на этого долбоеба. Я не знаю, он как ольгинец, просто я таких ебанатов только в политаче и ньюзаче видел. Такой отборный даун. Еще раз извиняюсь, братва.
Аноним 01/02/19 Птн 21:26:52 45895552
Что если создать коробку покрытую идеальным зеркалом изнутри, и поймать туда дофига световой энергии. А потом использовать такую коробку как аккумулятор бесконечной размерности. Ведь у света нет веса.

Какие подводные камни?
Аноним 01/02/19 Птн 21:36:31 45895753
>>458955
Гугол - излучение абсолютно черного тела
Аноним 01/02/19 Птн 21:53:15 45896254
>>458955
>Ведь у света нет веса.
>
Есть.
Аноним 01/02/19 Птн 22:21:43 45896955
Какого типа волны используются для передачи информации космос-земля. Вот например во время апполон-11 велась телетрансляция на землю. Какого типа волны использовались и какая у них пропускная способность?
Аноним 01/02/19 Птн 22:56:31 45897456
А что собственно мешает сделать нейросетку с количеством логических элементов как в мозге и с объёмной связью?
Слишком дохера и слишком сложно? На тех масштабах на которых пока могут работать лучшие чиповые производства/лаборатории, он выйдет слишком огромным?
Аноним 01/02/19 Птн 22:57:17 45897557
.png (57Кб, 1060x564)
1060x564
Уважаемые анонимные ученые с мировым именем, рассмотрите пожалуйста мою идею. Только что внезапно пришла мне в голову, я аж охренел от гениальности. Вроде я все верно продумал и система должна ускоряться. Тяга хоть будет небольшая, но для замены ионных движков сойдет. Конечно, если развить идею, то можно добиться существенной тяги.
Аноним 01/02/19 Птн 23:21:44 45897858
7b62f6b6b975ff9[...].jpg (5Кб, 170x170)
170x170
>>458975
>фотоны самоускоряются
Аноним 02/02/19 Суб 00:50:47 45898359
>>458978
> > фотоны самоускоряются
пфф

>>458975
> фотоны хотят
Аноним 02/02/19 Суб 12:56:16 45904260
>>458850
И вот, так на мой вопрос не ответили, а я ведь просто хотел узнать конкретное явление, которое уничтожит представление света как волны в некоторой среде. Так же меня удивляет, что вот для светоносной среды надо там что-то новое придумывать, а для современного почему-то нет. Но ведь уже есть газовая динамика. Чего там такого описывать нового? Очевидно, что есть сильное недопонимание меня в том числе. Вот есть газ, который когда-то считали идеальной жидкостью или идеальным газом, но не взлетело, газ этот обладает для нас почти идеальными свойствами, вторая скорость звука равна 300 000 000 мс. Вопрос, шо там такого то изобретать надо? Ау? Ответьте мне. Нахрена все эти нудные сложности с 4 измерениями, костыли для сложения скоростей? Что сложного в понимании очевидной информации - при скорости в 300 000 000 ВНЕЗАПНО ЛОБОВОЕ СОПРОТИВЛЕНИЕ ЭТОЙ СРЕДЫ ну не такое же как при 300 мс? Что сложного понять, что для атомов влияние этой среды сильнее и от того и их перемещение не строго линейное, а часто хаотичное? Что сложного в понятии присоединённый вихрь - электрон?

Заклинаю вас, напишите мне наконец те природные явления, которые просто топчут выше описанные предположения, для меня это крайне важно. И нет, я не тралль.
Аноним 02/02/19 Суб 13:05:30 45904361
aXm8707xjU.jpg (88Кб, 650x650)
650x650
>>458700 (OP)
Можно убрать татьяныча с главной страницы? пожалуйста
Аноним 02/02/19 Суб 13:56:52 45904862
>>459043
Да ладно тебе, пусть будет. Он же неплохо так деморализует и наводит апатию
Аноним 02/02/19 Суб 14:54:53 45905163
.gif (997Кб, 500x270)
500x270
Аноним 02/02/19 Суб 14:57:41 45905364
>>458974
Там вроде только научились в 60 слоев транзисторы фигачить, а тебе моск уже подавай.
Аноним 02/02/19 Суб 15:33:54 45906165
>>459042
Ничего не понятно, что ты хотел доннсти. Но если ты считаешь что-то костылями, это не значит что они ими являются.
Аноним 02/02/19 Суб 16:06:20 45906866
>>459043
>>459048
>>459051
Э, охуели? Я её нашёля, я не ебу кто это, но она(оно) сюда отлично подходит, она(оно) изображает что задумалась!! И никакой деморализации и апатии. И никуда она(оно) отсюда поэтому не уберётся, пошли нахуй.
Аноним 02/02/19 Суб 16:07:56 45906967
>>459061
А сколько их всего в мозге?
Как я понял один нейрон как один логический элемент рассматривать нельзя, а только как сразу кучу.
Аноним 02/02/19 Суб 19:06:19 45909868
>>459068
Картинка +/-, не сильно хуже или лучше многих других, а вот от самого треда чёт говнецом каким-то отдаёт, хз может у меня восприятие просто такое.
Аноним 02/02/19 Суб 19:44:54 45910269
>>459098
Ты ньюфажек? Это классический постоянный местный тред.
Аноним 02/02/19 Суб 19:53:07 45910470
>>459102
Это про тред с татьянычем было, в самом начале который.
Аноним 02/02/19 Суб 22:43:58 45912871
>>458700 (OP)
Подскажите плиз несколько признаков, которые есть у все людей и которых нет ни у одного животного. Только не надо всякую гуманитарную шляпу вроде способности в язык или самосознания, у дохуя людей этого нет. Моих знаний биологии хватило только на "человеческую" ДНК, что еще? Только должно быть гарантировано у всех, и у младенцев и у больных всеми возможными болезнями.
Аноним 02/02/19 Суб 23:21:42 45915572
>>458700 (OP)
Почему всякие там кошечки и собачки сначала во время течки сначала мурчат и призывно катаются по земле, а когда к ним подходит самец, начинают брыкаться?
Аноним 03/02/19 Вск 00:03:03 45917073
>>458700 (OP)
Так блэт, везде говорят что возраст Вселенной 13.8 млрд лет, однако радиус наблюдаемой Вселенной ~47 млрд лет. Как это блять возможно?
После Большого Взрыва Вселенная расширялась быстрее скорости света?
Аноним 03/02/19 Вск 00:08:32 45917174
Аноним 03/02/19 Вск 06:14:50 45920075
>>458700 (OP)
Насколько сильно зависит строение мозга от внешней формы черепа ? За счет чего, например, у некоторых индивидов формируется скошенный лоб ? И как это влияет на внутреннюю структуру мозга ?
Чем в плане строение отличается долихоцефальный и брахицефальный мозг ?
И почему у людей вне зависимости от расовой и этнической принадлежности так сильно отличается форма и размеры верхней части головы ? Например, всякие Арабы или Индусы встречаются как и с крошечным черепом и небольшим лбом, так и напротив, с огромным выпуклым лбом и шарообразной головой.
Есть какие нибудь интересные статьи и книги где затрагиваются подобные темы ?
Извиняюсь, если вопросы глупые, я дилетант и не скрываю это.
Аноним 03/02/19 Вск 06:56:30 45920276
Читал в Вики про испуг и наткнулся на это.
> У млекопитающих выделяют 4 стадии[2]:
> Убежать.
> Драться (если убежать не удастся).
> Оцепенеть (если противник явно сильнее).
> Потерять сознание (если оцепенение не помогло).
Почему так? Я вообще не врубаюсь, как так. Откуда это оцепенение взялось вообще? Отрыв башки просто. Я и раньше знал, что животные, люди в том числе, от страха цепенеют, но только сейчас осознал, как же это странно. Как это объясняется эволюционно?
Аноним 03/02/19 Вск 08:15:41 45920677
>>459202
мозг клинит и он не может выбрать что делать.
Аноним 03/02/19 Вск 11:36:36 45921178
>>459206
>мозг клинит и он не может выбрать что делать
Ну это ж и так понятно.
Он походу про то хули они все не передохли.

Олсо у людей понятно, давно уже идут селекции опущенных хуесосов, как в россии. У людей такие особи выживают неплохо.
А вот у животных? Где не "оцепенел-окейфэйс и пошёл работать и платить налоги опустившим тебя", а у животных "испугался что тебя съедят-оцепенел-тябя съели ещё легче, вообще не прикладывая усилий на догнать и борьбу"
Аноним 03/02/19 Вск 12:16:58 45921579
>>459171
Нам же втирают что быстрее скорости света ничего перемещаться не может?
Аноним 03/02/19 Вск 12:37:12 45921780
>>459215
В физике полно неудобных костылей, что поделать. То что тебя беспокоит на самом деле не 100% факт, а предположения, в реальности же большого взрыва не было, достоверный возраст вселенной, как и ее реальные размеры, никто не знает, также в реальности ничто и никто не запрещает двигаться со скоростями больше скорости света.
Аноним 03/02/19 Вск 14:31:07 45922881
>>458700 (OP)
Почему десятичное время до сих пор не ввели? Удобнее же будет для расчетов.
Аноним 03/02/19 Вск 15:07:53 45923282
>>459215
Распространение взаимодействия не может быть быстрее скорости света. На расширение пространства влияет что-то другое.
Аноним 03/02/19 Вск 16:13:01 45924083
p01h9j56.jpg (33Кб, 640x360)
640x360
>>459215
>Нам же втирают что быстрее скорости света ничего перемещаться не может?
а там и не перемещается
Аноним 03/02/19 Вск 16:24:43 45924384
>>459228
Придется перестраивать все эти расчеты под новую систему.
Аноним 03/02/19 Вск 17:57:55 45924785
>>459215
По пространству быстрее нельзя перемещаться. А само пространство может. Ты просто не слушай научпопоедов и местных обитателей. Лучше либо вообще ничего не знать, либо уже нормальные учебники изучать, а то задумываться начнешь.
Аноним 03/02/19 Вск 22:02:14 45927586
Что такое гендер по существу? Научен ли этот термин или шизоидный высер псевдоученых?
Аноним 03/02/19 Вск 22:33:42 45927887
>>459275
>Что такое гендер по существу?
Социальный пол. Допустимо ли тебе носить юбку, или "нимужык"?

>Научен ли этот термин
Да.
Аноним 04/02/19 Пнд 01:57:36 45930088
>>459275
> Что такое гендер по существу? Научен ли этот термин или шизоидный высер псевдоученых?
Постмодернисткий костыль
>Научен ли этот термин
Нет.
Аноним 04/02/19 Пнд 09:19:59 45930789
>>458700 (OP)
А помните был какой-то двигатель опровергающий закон импульса(или сохранения энергии , не ебу) от него еще у адептов оффициальной ноуки рвало жопу знатно, что там с ним?
Аноним 04/02/19 Пнд 10:04:03 45931490
>>459307
В космос вроде собираются запускать тестить. Или впервые, или опять, поздоровее образец.
Вроде так и не поняли откуда там берётся тяга, и опровергнуть замеры ошибками измерений тоже всё никак не могут.
Аноним 04/02/19 Пнд 10:52:25 45931591
если вся система сформировалась из продукта распада другой звезды
значит ли это что все из чего мы сейчас состоим горело ранее в одном месте
Аноним 04/02/19 Пнд 19:38:46 45935492
Почему, когда я слышу крики в кино или игре, мне похуй, а когда я слышу ирл крики во время каких-нибудь инцидентов, или смотрю видео с такими инцидентами, мне сразу становится понятно, что человек не притворяется и ему угрожает опасность? Это какая-то особая "паническая частота" голоса, или я просто сам не замечаю, как учитываю контекст?
Аноним 04/02/19 Пнд 20:10:07 45935593
Можно простыми словами объяснить почему двигаются литосферные плиты* Я прогуглил и у меня голова закипела, какая-то конвекция мантии. Какие-то потоки, я вообще не представляю себе какие могут быть потоки в твердых породах земли, это же не жидкости чтобы они так обменивались теплом, теплопроводимость камня же очень маленькая. Как это все происходит на пальцах?
Аноним 04/02/19 Пнд 20:12:51 45935694
>>459354
Естественно контекст. Мне тоже смотря кино абсолютно похуй а смотря реальные футажи каких-нибудь землятресений, смерчей реально крипово.
Аноним 04/02/19 Пнд 20:36:28 45935895
>>459355
представь куски глины на лаве - это самое доступное объяснение которое я могу дать
Аноним 04/02/19 Пнд 20:55:13 45935996
Аноним 04/02/19 Пнд 21:30:21 45936097
Нука объяснили мне бля нахуй, как это луч света в призме разделяется на разные цвета. Объяснили быра, че будет с одним единственным фотоном, запиздяченным в призму. Быра ска
Аноним 04/02/19 Пнд 21:56:02 45936298
тест
Аноним 04/02/19 Пнд 21:56:32 45936399
>>459360
учебник физики 7 класс раздел оптика
Аноним 04/02/19 Пнд 21:56:44 459364100
Стикер (191Кб, 500x500)
500x500
>>459360
>как это луч света в призме разделяется на разные цвета
Потому различные моды волны имеют разные фазовые скорости в оптичиских средах
>Объяснили быра, че будет с одним единственным фотоном, запиздяченным в призму
Приломится согласно его моде, т.е с классического на квантовый - какой энергии будет фотон.
Аноним 04/02/19 Пнд 22:04:08 459365101
какой сложности нужно создать кристалл, что бы фотоны задерживались в нем хотя бы на 1-2 секунды?
Аноним 04/02/19 Пнд 22:08:56 459368102
b69fd393bad18fd[...].jpg (36Кб, 500x427)
500x427
>>458700 (OP)
Почему скорость света 300к км/c, аллах так решил?
Аноним 04/02/19 Пнд 22:12:54 459369103
>>459355Представь яйцо. Яйцо - шар. Подвешенный в невесомости. И в вакууме. При температуре абсолютного нуля среды. У яйца внутри очень много много слоев. В самом внутри яйца горячее ядро, настолько горячее, что это аж плазма. Ядро это вращается вокруг своей оси, и оно заставляет двигаться и слои над собой. Теперь о самом верхнем слое. Он - остывший, он - твёрдый, это скорлупа-кора, и так как Земля вертится, а слои под корой все таки жидкие, то кора, обтираясь при вращении о свою атмосферу, подтормаживает изнутри магму. Ну и иногда магма короче за что то там изнутри коры запинается, и она своим вращением из за вращения земли вокруг своей оси потом натягивает за собой одну литосферную плиту на другую
Аноним 04/02/19 Пнд 22:14:35 459370104
>>459364Нихуя не понял, вот я фонариком в вакууме свечу - все фотоны пиздячут на призму с одинаковой скоростью, хули оно все равно на цвета делится?
Аноним 04/02/19 Пнд 22:18:38 459372105
>>459368
Потому что Эйнштейн больше не умел считать
Аноним 04/02/19 Пнд 22:22:40 459373106
>>459370
фотоны всегда пиздячут с одинаковой скоростью лол, и на призму и в призме и после.
Аноним 04/02/19 Пнд 22:47:50 459376107
>>459360
>Объяснили быра, че будет с одним единственным фотоном, запиздяченным в призму.
Единственный фотон запиздяченный в линзу,представляет собой волновой пакет (так как если бы у фотона была конкретная частота, то это давало бы полную неопределенность по его расположению, он был бы размазан по прямой). Но так как ты, как я понимаю, локализуешь всё-таки фотон, он имеет суперпозицию разных частот. Когда фотон пройдет через призму, его распидорасит, и его разные частотные составляющие полетят в разные сторон, но так как фотон один, то измеряя, ты найдешь его лишь в одном месте, будто бы он летел до этого имея конкретную частоту, если же прогонишь эксперимент много раз, сможешь увидеть состав этого ебучего изначального фотона. (надеюсь я не напиздел сильно, так как с фотонами никогда расчетов не вёл).
Аноним 04/02/19 Пнд 22:54:34 459378108
Откуда береться вращательное движение у звезд и планет? Я понимаю что это происходит еще на этапе зарождения, когда облака газа коллапсируют и начинают вращаться, но ПОЧЕМУ они начинают вращаться? Что является источником вращения?
Все что написано на вики:
>Самая распространенная теория объясняет это процессами, шедшими во времена образования планет. Облака космической пыли «сбивались в кучу», образуя зародыши планет, к ним притягивались другие более или менее крупные космические тела. Столкновения с этими телами и могли придать вращение будущим планетам. А дальше планеты продолжали вращаться по инерции.
Тоесть мы даже точно не знаем какого хуя все во вселенной вращается?
Аноним 04/02/19 Пнд 22:56:15 459379109
>>459378
>Тоесть мы даже точно не знаем какого хуя все во вселенной вращается?
Из-за гравитации, ебана
Аноним 04/02/19 Пнд 23:09:18 459381110
>>459368
Очевидно это задумка создателей, чтобы запереть лысых обезьян в гулаге "Солнечная система" навсегда + ограничить их развитие (ибо скорость компов и коммуникаций напрямую зависит от скорости света).
Аноним 04/02/19 Пнд 23:20:03 459383111
>>459379
Охуенное объяснение. А конкретней? Гравитация это хуйня что притягивает материю а не раскручивает её.
Аноним 04/02/19 Пнд 23:27:16 459385112
>>459383
Притягивает раскручивая, поскольку гравитация это искривление пространства.
Аноним 04/02/19 Пнд 23:31:23 459386113
>>459385
Пожалуйста не тролль его.
Аноним 05/02/19 Втр 00:01:31 459389114
Если два человека будут есть абсолютно одинаково и совершать абсолютно одинаковую физическую активность, то могут ди онт вообще иметь разный вес?
Аноним 05/02/19 Втр 00:34:51 459392115
>>459360
Если ты на автомобиле заедешь под углом на границу раздела асфальт-грязь, то жоповозку развернет не угол зависимый от скорости жоповозки на асфальте и на грязи. Это происходит от того что первое колесо вязнет в грязи и движется медленнее остальных колес. В точности то же самое происходит сфотоном, поскольку он частица (точнее система частиц), имеющая размеры.
Аноним 05/02/19 Втр 00:41:08 459393116
>>459386
Так что у нас действительно нет объяснения почему все крутится?
Аноним 05/02/19 Втр 00:57:13 459396117
>>459378
Очевидно, если облако газа немного вращалось, то будет вращаться звезда, сгустившаяся из него.
Аноним 05/02/19 Втр 01:00:11 459397118
>>459378
>какого хуя все во вселенной вращается?
Таковой ее сделали, очевидно же.
Аноним 05/02/19 Втр 01:54:55 459402119
>>459376
>представляет собой волновой пакет
Это классическое представление.
>он имеет суперпозицию разных частот
E = hv.
>Когда фотон пройдет через призму, его распидорасит
фотон очень стабилен и сечение γ -> γγ очень мало.
>если же прогонишь эксперимент много раз, сможешь увидеть состав этого ебучего изначального фотона
Сделай это для лазера.
Аноним 05/02/19 Втр 02:42:45 459411120
>>459211
Это оцепенение происходит по сути когда хищник вот вот и бросится на тебя. Поздно бежать, а сопротивляться возможностей нет. Просто шок, но отойти от него у тебя уже времени банально нет.
Аноним 05/02/19 Втр 02:42:51 459412121
>>459402
>Сделай это для лазера.
У лазера очень маленькая дисперсия частоты. Там все фотоны как на подбор. Какой смысл в этом? "Пролетит" под своим углом каким в оптике проходит и всё. Хотя если попробовать бахнуть через дифракционную решетку, можно наверное замерить эту дисперсию и разделить фотоны.
>E = hv
А к чему ты это? Я просто говорю что в луче разное говно может быть. Просто случай фотона с числом заполнения 1 это странно даже.
Аноним 05/02/19 Втр 02:46:34 459413122
>>459396
Так блять вопрос почему это облако газа началось вращаться?
Аноним 05/02/19 Втр 02:50:27 459414123
>>459413
вращение это постоянное падение в яму, роль которой выполняет центр тяжести. Но постоянное падение мимо. Как луна, которая постоянно падает на землю, но постоянно промахивается, образуя стабильную орбиту таким образом. Будь земля же плоской, то давно бы попала.
Аноним 05/02/19 Втр 02:50:42 459415124
>>459413
Флуктуации видимо. Момент то вроде пока сохраняется.
Аноним 05/02/19 Втр 02:53:51 459417125
>>459415
Да не, свойство гравитации и пространства. Достаточно образоваться небольшому центру тяжести, как он начнёт вынуждать всё вокруг себя вращаться. Более тяжёлые объекты, такие как планеты и звёзды этот момент вращения и сохраняют, и после того как окончательную форму примут. Без вращения банально ничего образоваться не может в нашей вселенной.
Аноним 05/02/19 Втр 02:55:21 459419126
загружено (8).jpg (12Кб, 262x193)
262x193
>>459413
Искривление пространства гравитацией, сказали же тебе. Объект не может двигаться по прямой в искривленном пространстве.
Аноним 05/02/19 Втр 02:57:11 459420127
>>459417
>Да не, свойство гравитации и пространства
Ты траллишь? Посмотри в уравнения ОТО, они симметричны.
>Достаточно образоваться небольшому центру тяжести, как он начнёт вынуждать всё вокруг себя вращаться.
Момент сохраняется. Алё. Оно визуально просто вращаться быстрее начинает, но изначально момент был, вопрос именно в этом изначальном моменте.
>>459419
Бля хватит траллить. Заебал. Косплей петухов наевшихся научпопа надо делать тоньше.
Аноним 05/02/19 Втр 03:00:50 459421128
>>459420
>Бля хватит траллить. Заебал
Никакого траллинга, ебана.

Вот простейший опыт
https://www.youtube.com/watch?v=EIEOGoBA4FA

Частицы газа начнут вращаться из-за гравитационной ямы.
Аноним 05/02/19 Втр 03:01:52 459422129
>>459420
Началось это по всей видимости примерно с самого начала вселенной. Но тут не важно, запусти просто тонны газа равномерно, что бы его хватило на какие-либо структуры и он мог гравитировать, и через какое то время и в этом пространстве без всяких флуктуация и БВ всё начнёт вращаться. Ибо без вращения ничего образоваться не может физически. Гравитация заставляет объект вращаться вокруг себя, он разгоняется и ударяется в центр тяжести, передавая и ему свой импульс, в итоге разгоняя и его скорость вращения
Аноним 05/02/19 Втр 03:06:57 459424130
>>459422
Думаю как раз таки в больших объемах вселенной момент близок к нулю. Это видимо просто флуктуации.
Аноним 05/02/19 Втр 03:07:25 459425131
Стикер (127Кб, 512x382)
512x382
>>459421
>начнут
А до этого они не вращались видимо.
Аноним 05/02/19 Втр 03:09:32 459426132
151281536514429[...].png (255Кб, 1600x814)
1600x814
>>459424
Да в больших объёмах во вселенной по всей видимости всё близко или равно нулю. Вселенная в суперпозиции, она вроде бы и есть, но её значение общее равно нулю.
Аноним 05/02/19 Втр 03:14:17 459427133
>>459412
Дак речь шла о фотоне. Фотон не может быть пакетом разных частот. Он просто E = hv. v - частота.
А вот для лазера да, там каждый фотон как на подбор.
>Единственный фотон запиздяченный в линзу,представляет собой волновой пакет
К этому претензия была.
Аноним 05/02/19 Втр 03:17:03 459428134
>>459420
Он и не траллит, он такой
Аноним 05/02/19 Втр 03:29:54 459429135
>>459427
> этому претензия была.
Ну если у пакета суммарное заполнение 1. То в чём проблема? Фотон может иметь суперпозицию спиральности, что ему мешает иметь суперпозицию в частотах?
Аноним 05/02/19 Втр 03:48:59 459430136
>>459429
>что ему мешает иметь суперпозицию в частотах
Это конечно заебись и ничто не мешает, но это заебно и нужны разные фотоны и нелинейная среда. Если ты пускаешь один фотон в призму, то не распидорасит.
Аноним 05/02/19 Втр 04:18:40 459431137
Не знаю в какой доске спросить. Вопрос по химии полимеров. Скажите, пожалуйста, из какого полимера сделана белая посудина от доширака в которой собственно доширак и заваривается? Это таки полипропилен? Я правильно понимаю?
Аноним 05/02/19 Втр 04:24:46 459432138
>>459431
И второй вопрос касаемо полипропилена. Написано, что у полипропилена температура размягчения минус 20... минус 10 С.

Правильно ли я понял, что при 0 градусов он твердый? А при минус 20 начнется размягчаться что ли?
Аноним 05/02/19 Втр 05:17:35 459434139
Создаётся ли новая нейронная связь каждый раз после того, как я посру?
Аноним 05/02/19 Втр 06:33:34 459435140
>>459434
Только если после съесть, что высрал.
Аноним 05/02/19 Втр 07:42:33 459438141
>>459411
Ну и чё ты написал? И так понятно что это, вопрос как такая хуйня закрепилась эволюционно у кого-то кроме людей, ведь очевидно что выживаемость у тех кто до конца пытался убежать или сопротивляться будет выше.
Аноним 05/02/19 Втр 08:11:41 459440142
Жизнь образовалась одним видом организмов или сразу несколько было? Или второй вид образовался, когда мутировавший представитель первого случайно попробовал говно товарища и оно ему понравилось, затем третий у второго и так по нарастающей?
Аноним 05/02/19 Втр 08:46:30 459445143
Верно ли, что если б метеорит ёбнул лет 120лямов назад, т.е. задолго до кризиса динозаврией биоты, то они вполне могли бы и не вымереть и даже бы скорее всего не?
Аноним 05/02/19 Втр 09:42:05 459453144
Что можно читнуть по квантовой механике, ее интерпретациям, связи с макромиром из современного?
Аноним 05/02/19 Втр 10:27:17 459463145
>>459438
на 0.01%? травоядные для выживаемости плодятся как кролики, либо большие и ходят кучами.
Аноним 05/02/19 Втр 10:51:19 459472146
>>459453
Линейная алгебра, спектры и функциональный анализ.
Сам процесс перехода с микромира на маркомира именно в них количественно описывается. Не читай никакой философской хуиты, она все равно бесполезна.
Аноним 05/02/19 Втр 11:16:02 459474147
>>459472
>Два основных раздела математики
Лол, очевидно
Только я думаю, что вопрос заключался в
>посоветуйте что-нить простенького, чтоб прочитать и понять типа, ну чтоб поменьше этих закорючек и чтоб поновее
А ты тут со своей математикой, блядь, охуеть блядь
математик
Аноним 05/02/19 Втр 11:49:55 459480148
>>459474
Как правило многие даже базового раздела математики не осиливают, а лезут блядь разбираться в физическую говнину.

В любом случае даже классическая физика строиться на сложных математических конструкциях, а не хуйне ну вот это точка типо двигается. Все просто и понятно не получится объяснит. А электродинамика для многих вызывает инфернальных ужас, хотя что там подумаешь, что там сложного?

И в вопросе по интерпретации квантовой теории, как на пальцах объяснить что такое линейные операторы на бесконечномерных пространствах, что такое их собственное значение, что такое свертка? А на этом строиться вся квантовая теория, и я еще не видел интерпретации этих вещей, хотя они вполне заслуживают это.
Аноним 05/02/19 Втр 11:56:42 459483149
>>459480
>как объяснить операторы n-мерных порстранстф светка кванты
А хуй знает, я не объясняю ничё, я прастой мужик-числоёб, по банаху живу, да встаю рано. На пробежку во
Аноним 05/02/19 Втр 11:57:50 459485150
>>459480
Ну они хотят и рыбку съесть и нахуй сесть. А так как правило не получается.
Аноним 05/02/19 Втр 12:03:37 459486151
>>459485
Они... Послушай, Джонни, они нашли меня. Они знают, где я, я больше не могу, Джонни, я больше не могу этого выносить! Эти..эти физики.. они хотят больше, понимаешь? "По-новой давай, нужен другой аппарат!". Они мучают меня, Джонни! Я не спал четверо суток.. Вчера они мне сказали, что заберут мою рыбку. Они хотят сесть на хуй, Джонни! Их надо остановить!!!
Аноним 05/02/19 Втр 13:15:14 459487152
>>459463
>0.01%
Ты скозал? В любом случае этого бы хватило.
Аноним 05/02/19 Втр 13:30:34 459488153
>>459487
не хватило бы и близко. ты как то не понимаешь как работает отбор. отбор идёт из тех, кто лучше ебался, а не из тех, кто дольше жил.

Тут речь уже обычно о старых, больных, отставших от стада. И без стада они совершенно не представляют как себя ввести. От хищников они так же привыкли бегать в стаде.

Более резвый козёл тут всё равно имеет практических никаких шансов на долгое выживание, так и какая разница, если он окажется достаточно резвым. Он уже никого не выебет. Но ловким и резвым в такой ситуации не будет. Ибо раз он попал так, то это говорит лишь о том, что с ним что то не так.

Да я и сейчас погуглил, там у козлов речь вроде бы вообще о генетической болезни, когда у них в случае опасности начинается паралич секунд на 15. Как видишь этому гену ничего не мешало расплодиться и выжить. Ибо кого волнует что он так тупо помер без сопротивления, если он уже кого то выеб.
Аноним 05/02/19 Втр 13:34:14 459489154
>>459488
Впрочем выжил этот ген и расплодился скорее всего благодаря человеку. Который спаривал их не обращая внимания на такой пустяк, как паралич при выбросе адреналина или каких других гормонов. Фермеру это никак не мешало.
Аноним 05/02/19 Втр 13:56:43 459490155
>>459489
Выше ты накатал полную хуйню, а вот это годна мысль.
Ген закрепился у тех видов, которые ловили, но не убивали/следали, а начинали разводить.
Легче поймать>больше таких будет поймано>среди разводимых их будет больше, и пошло-поехало.
Аноним 05/02/19 Втр 14:20:42 459491156
>>459373
Так хули они разным цветом светятся после призмы тооо?
Аноним 05/02/19 Втр 14:22:28 459492157
>>459491
Они и до призмы с разным цветом пиздячили
Аноним 05/02/19 Втр 14:30:32 459494158
>>459431Там маркировка должна быть, посмотри в магазине
Аноним 05/02/19 Втр 14:31:52 459495159
>>459492Ненене, вот луч света от фонаря, он белый, а от призмы радуга.
Аноним 05/02/19 Втр 14:37:38 459496160
Если в огромном зоопарке поселить представителей всех видов организмов всех эпох, кто победит если какие-то простейшие/вирусы, то отбросываем их, интересны только многоклеточные?
Почему океан относительно пустой и не кишит тварью как сельдь в бочке? Это же питательный рассол по сути.
Аноним 05/02/19 Втр 14:37:48 459497161
>>459495
>луч света от фонаря, он белый
нет белых фотонов, это смесь цветов.
Аноним 05/02/19 Втр 14:38:15 459498162
>>459392 хорошо, понял, спасибо.
Аноним 05/02/19 Втр 14:42:27 459499163
Аноним 05/02/19 Втр 14:49:20 459500164
>>459499
>Люди
Люди представляют собой что-то только благодаря сохранённой с каменных веков информации и её передаче.
Обнули её-и они соснут почти всем.

Вообще это очень забавно, пропуск всего одного поколения, и всё, будто ничего и не было.
Аноним 05/02/19 Втр 14:53:26 459501165
>>459496
>Если в огромном зоопарке поселить
Всё зависит от условий в этом зоопарке.
Если постоянные, то какие они?
Изменяющиеся? В таком зоопарке ты и живёшь.
Аноним 05/02/19 Втр 15:25:43 459503166
>>459490
Выше я писал сначала об одном, а после затронул тему козлов уже. Ты писал об оцепенении от страха, у них же совсем другое.Паралич от перевозбуждения, а не просто испуга. Они когда играют стабильно падают так же.
В дикой природе этого гена нет. Он изначально селекционный. Точнее мог и появится, но не выжить там.

К замиранию от страха это же никакого отношения не имеет всё.
Аноним 05/02/19 Втр 17:55:51 459522167
>>459472
>Линейная алгебра и функциональный анализ.
Спасибо, это все было в универе. И, ты не поверишь, сама квантовая механика была.
>спектры
И что это? Раздел математики? А книга какая?
>Сам процесс перехода с микромира на маркомира именно в них количественно описывается.
НЕ МОЖЕТ БЫТЬ!
>Не читай никакой философской хуиты, она все равно бесполезна.
Я вот струны считаю бесполезной хуитой. Но я же не трублю об этом на каждом шагу.
>посоветуйте что-нить простенького, чтоб прочитать и понять типа, ну чтоб поменьше этих закорючек и чтоб поновее
Скорее не простое, а именно мнения разных ученых по этому поводу. Ну и да, я понимаю, что к науке это должно иметь не самое прямое отношение


Аноним 05/02/19 Втр 22:42:28 459537168
>>459522
Подожди если у тебя был линал и функан, то вопрос
>И что это?
Звучит очень странно. Кажется ты нас наебываешь.
Аноним 05/02/19 Втр 23:01:34 459548169
Аноним 05/02/19 Втр 23:29:14 459550170
>>459537
Хорошо, я тебя наебываю, господин параноик. Заходишь сюда чисто попиздеть, да? Понятно, что я мог уже двести раз нагуглить что-нибудь по своей проблеме. Но прикинь, я правда рассчитывал, что мне первым же постом ответят по теме. Совсем ебанутый, да?
05/02/19 Втр 23:56:02 459554171
>>459550
Ну в общем совет такой. Совет, мне кажется,правильный, нужный. Честный такой совет. Что пока ты разберешься в математике, тебе уже пофиг будет. Ты станешь математиком лол. Потому что это интересно. А потом км интересует? Ну я не знаю. Просто ради интереса, о чем и совет — дерни за мудя вузовского физика. Рандомного. Я раньше просто ходил вольнослушателем. Это бесплатно. Спроси его о книгах, задай вопросы. Поймай студента и спроси. Так все делают. Почти никто там не понимает.
А и да. Спрашиваешь про книги. Ну. Sci уже сдох. Правда, тут прикольно обмазывать говном вучоних, когда настроение плохое. Но спорить?! СПРАШИВАТЬ??!!! Бесполезно, я понимаю, что сам себя обосрал) Но ты послушай. Я типа мудрую вещь написал выше. Лучшее объяснение — живое, доходчивое и образное, без сухих формул пока что, да и лучше запомнишь. peace
Аноним 06/02/19 Срд 02:53:44 459564172
Есть такой сайт неплохой, mathprofi.ru, там мужик очень просто обьясняет практическую часть картофельного анализа, картофельной геометрии и тд - интегралы, пределы, прямые в пространстве и вот это все
Есть что-то подобное по физике, чтобы научиться решать задачи? Интересует механика и мкт. Сам не нагуглил
Если нет, то что посоветуете небольшое по обьему, подходящее под мои цели? Времени у меня до конца февраля +-, могу часа 3 в день ботать наверное + часов по 5 на выходных
Аноним 06/02/19 Срд 05:53:25 459571173
Могут ли ЭМ волны радиодиапазона влиять на здоровье человека?
Аноним 06/02/19 Срд 06:12:43 459572174
>>459571
Несут энергию - значит могут. Сожженные на лету птицы, попавшие в главный лепесток загоризонтников тому пример.
Аноним 06/02/19 Срд 06:32:02 459573175
>>459572
Окей, а есть какие-то исследования, подтверждающие их вред для человека? Я знаю, что существуют допустимые нормы для напряжения магнитного поля, превышение которых ведет к ухудшению здоровья, но не слишком хорошо понимаю, как именно это перекликается.
>Сожженные на лету птицы, попавшие в главный лепесток загоризонтников
Пруфы есть?
Аноним 06/02/19 Срд 15:20:00 459596176
>>459573
На здоровье в понимании желтой прессы - нет, не несут. Несут только так же как кипяток. Надо мощную стоячую волну в небольшом пространстве, чтобы тебе как то навредить - поджарить. Ни один бытовой прибор не способен этого сделать, если конечно в микроволновку не залезешь.
Только ультрофиолет и выше может нанести вре здоровью.
Аноним 06/02/19 Срд 15:24:30 459598177
Аноним 06/02/19 Срд 16:16:35 459606178
Возможна "обратная дисперсия"? Тоесть если я буду хуячить разноцветными лучами на призму, соберется ли он в белый?
Аноним 06/02/19 Срд 16:52:02 459609179
>>459606
Если ты пропустишь посты из /b обратно через роутер они станут чистой информацией? Нет, останется куча дерьма.
Аноним 06/02/19 Срд 17:07:16 459610180
>>459606
>если я буду хуячить разноцветными лучами на призму, соберется ли он в белый?
Конечно, волновые явления, по определению - обратимы.

И не слушай дегенератов
>>459609
рассуждающих по аналогии.
Аноним 06/02/19 Срд 17:09:43 459611181
Аноним 06/02/19 Срд 17:24:47 459613182
пруф хоть один что это возможно
Аноним 06/02/19 Срд 17:36:25 459614183
>>459613
призму разверни
пусти разложившуюся радугу обратно в призму
Аноним 06/02/19 Срд 17:38:18 459615184
>>459614
ясно, другого и не ожидал.
Я на словах и в будущее смогу путешествовать.
Аноним 06/02/19 Срд 18:02:07 459617185
>>459615
бля, посмотри что такое хроматическая абберация и как с ней справляются.
Аноним 06/02/19 Срд 18:14:10 459618186
>>459617
я не знаю что ты мне втюхать пытаешься, еще скажи что мкс на самом деле на орбите находится.
Раз это так просто, так скинь ну видео с записью эксперимента? что сложного?
Аноним 06/02/19 Срд 18:18:54 459619187
>>459618
Вообще-то свет из вакуума в воздух попадает и раскладывается на радугу, и мы бы ничего не видели, а мкс летает и собирает их снова в стопку, поэтому свет на земле белый. Раньше чтобы что-то увидеть, надо было ч/б фильтр применять, поэтому все фотки черно белые, а картины цветные - это воображение, поэтому художники считались ненормальными, у них на самом деле просто мозг мог свет в пучки обратно собирать.
Аноним 06/02/19 Срд 19:18:08 459623188
empty.png (0Кб, 200x200)
200x200
>>459613
Посмотри на пикрелейтед. ПОДУМОЙ.
Просветление будет вообще охуешь, обещаю.
Аноним 06/02/19 Срд 19:57:40 459626189
То есть пространство на самом деле не искривлено? Почему? И почему возник якобы миф, что оно искривлено?
мимошел
Аноним 06/02/19 Срд 20:18:43 459630190
двач как всегда обосрался.
Говорю раз реально - дайте видео эксперимента с призмой
РЯЯЯЯЯ ПОДУМОЙ ПОДУМОЙ РЯЯЯЯЯ
Аноним 06/02/19 Срд 20:50:16 459631191
>>459630
сам то сделай и докажи нам всем
Аноним 06/02/19 Срд 21:10:17 459634192
>>459626
Искривлено. потому что может. ну как еще назвать пространство где кротчайшие расстояние это не прямая линия? И да, это измеряемо.
Аноним 06/02/19 Срд 21:21:45 459635193
>>459630
>дайте видео эксперимента с призмой
Блять, ты заебал, долбоеб
Отчего свет в призме "разлагается" (то бишь происходит дисперсия) блять? Из-за разности длин волн компонентов белого света, а отсюда и из-за разности коэффициента преломления. Один квант переломится как ему и следует в зависимости от частоты ЭМ излучения
Какой тебе нах эксперимент нужен?
Аноним 06/02/19 Срд 21:31:18 459636194
>>459635
ЭТО ВЫ а не Я утверждал, что если пропустить радужный цвет обратно через призму он станет белым лучом. И спросил где пруфы. Пошли какие то маниотмазки на уровне НУЖНО ПРОСТО ВЕРИТЬ
Аноним 06/02/19 Срд 21:35:44 459637195
>>459636
То есть ты хочешь сказать, что процесс необратим? Разложение не симметрично во времени?
Аноним 06/02/19 Срд 21:50:10 459639196
hqdefault.jpg (7Кб, 480x360)
480x360
>>459636
Будь ты троль или просто тупой, который в школе прогуливал геометрическую оптику, можешь пездовать отседова, если тебе не нравятся ответы. У физички в школе спросишь тогда.
Аноним 06/02/19 Срд 21:51:42 459640197
Ясно, 0 пруфов, один пиздеж.
Аноним 06/02/19 Срд 21:57:53 459641198
867567456234345[...].jpg (44Кб, 878x577)
878x577
>>459615
Вот те описание эксперемента, сам только делай, у меня денег нет на призмы и зеркала.
Аноним 06/02/19 Срд 22:02:58 459643199
>>459641
значит даже на том же ютубе, где есть все что угодно. Нет записи с этим опытом, серьезно?) Что вы тут пытаетесь втереть? на деле такое работать не будет.
Аноним 06/02/19 Срд 22:10:10 459645200
Стикер (0Кб, 464x512)
464x512
>>459643
Запиши видео, будешь первым, Нобелевку получишь. Я тебе лишь изи мани идею дал.
Аноним 06/02/19 Срд 22:28:35 459647201
Аноним 06/02/19 Срд 22:28:40 459648202
>>459645
Как дурак по комнате бегать фотоны цветные ловить? Вы за кого меня держите?
Аноним 06/02/19 Срд 22:51:55 459651203
>>459647
Ну и кто из нас на хуй пойдет? На выходе он получил такую же радугу, но более сфокусированную, чем от одной призмы. Исходного одного луча не получилось.
Аноним 06/02/19 Срд 23:13:57 459654204
>>459636
>>459643
https://www.youtube.com/watch?v=uucYGK_Ymp0
А долбоебов как всегда банят в англоязычной части ютуба ну или это потому что на 7м классе церковно-приходской инглиш в принципе не изучается
Аноним 06/02/19 Срд 23:15:03 459655205
>>459654
Ну и кто из нас на хуй пойдет? На выходе он получил такую же радугу, но более сфокусированную, чем от одной призмы. Исходного одного луча не получилось.
Аноним 06/02/19 Срд 23:17:36 459656206
>>459655
https://www.youtube.com/watch?v=yy-vAc6hPig
Теперь вручную предлагаешь добиться сверхточной фокусировки? Из шараги своей для начала выпустись, чтоб такими вопросами задаваться
Аноним 06/02/19 Срд 23:21:52 459657207
>>459656
для оптики и аппаратуры луч света начальный будет лучом света, а то что получилось в итоге микрорадуга. Не луч. Спектр цветов.
Доказательств обратных никто из вас не предоставил.
Аноним 06/02/19 Срд 23:27:31 459658208
Стикер (127Кб, 450x450)
450x450
>>459657
>луч света начальный будет лучом света
лол
Аноним 07/02/19 Чтв 01:21:29 459662209
>>459613
>>459623
Вот этого двачую. Возьми лупу побольше, и посмотри внимательно.
Аноним 07/02/19 Чтв 01:25:42 459663210
>>459623
Я вижу цветные огоньки: красный, зеленый, синий и их очень много.
Аноним 07/02/19 Чтв 01:34:40 459665211
>>459626
>И почему возник якобы миф, что оно искривлено?
Пошло от в глаза ебущихся неликвидовых геометров, называющих кривые линии прямыми.
Аноним 07/02/19 Чтв 01:45:42 459667212
>>459626
Эйнштейн когда своё уравнение делал, то оставил сначала константу, которая фактически отвечала за кривизну пространства (выпуклое или вогнутое) и т.п. Но подумал мол "что за хуйня, не красиво, где симметрия" и выпилил. А потом астрономы заметили расширение вселенной и такие "ебать, там константа".
Аноним 07/02/19 Чтв 01:52:27 459668213
Есть константа скорости движения в вакууме для безмассовых частиц - факт.
Частицы, имеющие массу, могут только стремиться к этой сокорости, затрачивая на это огромную энергию, и даже вся энергия вселенной не разгонит частицу до с - тоже факт.
Но - почему? Почему частицы с массой "цепляются" за пространство при попытке их разогнать? Что заставляет фотоны двигаться со скоростью с, а не с+1? Что их стопорит?
Аноним 07/02/19 Чтв 01:59:20 459669214
Аноним 07/02/19 Чтв 02:15:58 459670215
>>459668
>Почему частицы с массой "цепляются" за пространство при попытке их разогнать?
Так главное же не просто цепляются, а сопротивляются только дополнительному разгону, а уже полученый-сохраняют.
Аноним 07/02/19 Чтв 02:31:29 459671216
Почему "безмассовые частицы" безмассовы? Кто это выдумал? Не может же частица совсем не иметь массы.
Аноним 07/02/19 Чтв 03:40:52 459672217
>>459671
Ну так про массу покоя же говорят.
Аноним 07/02/19 Чтв 03:44:17 459673218
>>459672
еще один с массой покоя, нука что это за зверь такой?
Аноним 07/02/19 Чтв 03:58:58 459674219
>>459671
Потому что они не взаимодействуют с полем Хиггса, который массу частицам и создаёт.
Аноним 07/02/19 Чтв 04:04:12 459675220
>>459355
Верхний слой мантии жидкий как кисель. По нему куски коры и плавают.
Аноним 07/02/19 Чтв 04:06:33 459676221
>>459355
Деление на твёрдое и жидкое весьма условно. Возьми куб метала и он постепенно растечётся как жидкость, хоть и займёт это может дестяки, сотни тысяч или может миллионы лет. Так если и смотреть на кору земли в ускоренном режиме очень, то она и будет ввести себя как жидкость по факту.
Аноним 07/02/19 Чтв 04:15:43 459677222
>>459673
Это E/c^2, где E это полная энергия частицы в системе отсчета связанной с частицей. В классической физике, этой величине соответствует обычная масса.
Аноним 07/02/19 Чтв 04:29:50 459678223
>>459677
А какая еще масса бывает тогда релятивиской нет.
Аноним 07/02/19 Чтв 04:33:24 459679224
>>459678
>А какая еще масса бывает тогда
Только такая. Понятие релятивистской есть, но от неё отошли так как неудобная она. Говорят просто про энергию.
Аноним 07/02/19 Чтв 04:36:49 459680225

>>459676
У тебя каша в голове, твёрдое это твёрдое. А ты говоришь об аморфных веществах, как смола, шоколад, стекло, ...
Аноним 07/02/19 Чтв 04:38:54 459681226
>>459679
Ну тогда и масса "покоя" тоже не нужна, просто масса лол.
Аноним 07/02/19 Чтв 04:43:51 459682227
>>459680
Аха, стекло аморфное вещество, при том, что многие металлы растекаются быстрее.

И аморфное вещество относится к твёрдым. Твёрдое это твёрдое, но на больших промежутках времени она будет растекаться ровно так же. Это аналогия была и что неверно относится к твёрдым телам, как к не изменчивым на масштабах в миллионы лет.
Аноним 07/02/19 Чтв 04:44:22 459683228
>>459681
Просто масса тоже не нужна.
Аноним 07/02/19 Чтв 06:21:59 459684229
>>459651
Нахуй идёшь ты, потому что видео даже до конца не досмотрел.
Аноним 07/02/19 Чтв 06:52:29 459685230
>>459682
Твёрдое тело это твёрдое тело, кристаллическая решётка, никуда оно не растекается. Может деформироваться, да, но твёрдый кристалл нельзя рассматривать как что-то жидкое, никак вообще, по определению. Твёрдое вещество настолько же жидкость, насколько оно газ. Это же очевидно.

А литосферные плиты двигаются как раз потому, что астеносфера, на которой они лежат, очень даже жидкая.
Аноним 07/02/19 Чтв 08:11:07 459686231
>>459685
>Твёрдое тело это твёрдое тело, кристаллическая решётка, никуда оно не растекается. Может деформироваться, да, но твёрдый кристалл нельзя рассматривать как что-то жидкое, никак вообще, по определению. Твёрдое вещество настолько же жидкость, насколько оно газ. Это же очевидно.
Повышаешь температуру-оно плавится, повышаешь давление-твердеет.
А при дохулионных значениях одного и второго?
Аноним 07/02/19 Чтв 08:21:59 459687232
>>459685
>газ и жидкость - это совершенно разные вещи, это же очевидно даже младенцу!
>что вы мне про сверхкритическую жидкость втираете?
>в какую дурку?
Аноним 07/02/19 Чтв 09:40:43 459691233
image.png (146Кб, 800x541)
800x541
>>459686
>А при дохулионных значениях одного и второго?
Критическая жидкость либо твердое тело
Аноним 07/02/19 Чтв 09:49:06 459692234
>>459691
Для какого вещества пик?
Аноним 07/02/19 Чтв 09:49:27 459693235
Аноним 07/02/19 Чтв 10:25:53 459694236
>>459691
Не все фазы подписаны.
Аноним 07/02/19 Чтв 14:30:35 459711237
>>459685
Ничего она не жидкая, а очень даже твердая, охуеть какая твердая, при таком то далвении. Как раз при таком давлении она еще станосится достаточно вязкая, чтобы были все чудеса литосферных перемещений.
Аноним 07/02/19 Чтв 14:35:15 459713238
>>459686
Астеносфера жидкая, потому что высокая температура. Мантия ниже твердая, потому что высокое давление. И что?

Изначально говорилось о том, что твёрдое вещество в обычных условиях ведёт себя как жидкость в длительной временной перспективе, это же ахинея. Ты не согласен?
Аноним 07/02/19 Чтв 14:37:03 459714239
>>459711
Магма это твёрдое вещество, я правильно тебя понял?
Аноним 07/02/19 Чтв 14:50:43 459716240
>>459714
Это газообразное текучее твёрдое вещество. Ещё вопросы, неуч?
Аноним 07/02/19 Чтв 14:57:07 459718241
>>459714
Не магма, а мантия. Магма это то что на поверхность выходит.
Мантия очень твердая, но достаточно вязкая очень мало из-за давления.
Аноним 07/02/19 Чтв 15:11:06 459720242
Можно ли научиться шевелить по отдельности каждым пальцем на ноге, или это технически невозможно?
Аноним 07/02/19 Чтв 15:21:56 459723243
>>459720
Можно вращать пальцами на ногах по отдельности и еще хером вертеть, зависит от того на сколько ты сконцентрирован и прокачен мозгом. Очень много мышц можно по отдельности контролировать.
Аноним 07/02/19 Чтв 22:05:13 459766244
images (4).jpg (6Кб, 192x263)
192x263
Почему еще никто не использовал квантовую телепортацию для сверхсветовой связи?
Аноним 08/02/19 Птн 00:37:39 459774245
>>459766
Я использовал. Общался с прищельцами.
Аноним 08/02/19 Птн 02:33:22 459777246
>>459685
растекается и испаряется. кристалическая рещётка тут не обязательно более устойчива и крепка, чем аморфное состояние.
Аноним 08/02/19 Птн 02:48:21 459778247
Нет ли надежд на создание метастабильного водяного льда, который останется твердым при нормальных (+20 и атмосферное давление) условиях?
Аноним 08/02/19 Птн 05:06:13 459782248
blob (23Кб, 662x527)
662x527
>>459766
Потому что два фотона, летящие в разных направлениях со скоростью v1 = v2 = c
- равноудаляются друг от друга со скоростью света. Пикрил.
Аноним 08/02/19 Птн 10:40:48 459797249
>>459782
Мне кажется, или это порождало бы кучу парадоксов ирл?
Аноним 08/02/19 Птн 11:00:08 459800250
>>459797
Да. Фонарики стоили бы дешевле.
Аноним 08/02/19 Птн 11:19:40 459801251
>>459782
Причем тут твои фотоны, речь о квантовой запутанности.
Аноним 08/02/19 Птн 12:21:13 459803252
>>459801
А квантовая запутанность не передает информацию.
Аноним 08/02/19 Птн 12:39:13 459805253
>>459803
Но ведь есть слабые измерения не разрушающие запутанность
Аноним 08/02/19 Птн 12:58:48 459807254
>>459805
Да, но чтобы сохранить полностью состояние, тебе придется замерить с бесконечной погрешностью, либо замерить и стереть свои данные, прежде чем их кто-то мог бы увидеть.
Аноним 08/02/19 Птн 13:08:12 459809255
>>459805
Суть квантовой запутанности в том, что прикладываем какие-нибудь разные операторы на ОДНО состояние квантовой системы из Н-частиц, в итоге мы получаем это же ОДНО состояние системы из Н-частиц в независимости от типа операторов. Из этого состояния мы никак не можем опередить какой-именно был оператор через другой оператор.
Аноним 08/02/19 Птн 17:04:56 459839256
Как чёрные дыры сохраняют момент вращения? Ведь он же проявляется только вследствие неравномерности распределения массы в теле, а информация о нём полностью теряется в ходе образования чд.
Аноним 08/02/19 Птн 17:06:07 459840257
>>459839
сверхновым это скажи, мальчик
Аноним 08/02/19 Птн 18:23:25 459843258
>>458906
Другой анон, ты пишешь бред хотя бы по той причине, что никакого ИИ не существует
Аноним 08/02/19 Птн 20:14:45 459850259
>>459797
Гугли что такое гиперболическое вращение и гиперболическая ортогональность.
>>459801
То что квантовая запутанность это не форма перемещения, тебе чтобы передать квантово запутанный фотон надо его послать как раз со скоростью света лол. Это так не работает, как некоторые себе уже придумали.
Аноним 08/02/19 Птн 22:36:35 459855260
>>459839
>Ведь он же проявляется только вследствие неравномерности распределения массы в теле
Момент это более фундаментальная вещь на самом деле. Эта хуйня выстреливает в ОТО, там тоже сохраняется момент. Эта хуйня выстреливает в квантах, там момент вообще какая-то ебучая магия.
Динамика Аноним 08/02/19 Птн 23:49:40 459860261
Какую информацию мне должен давать факт того, что тела движутся друг относительно друга без проскальзывания?
Аноним 09/02/19 Суб 00:12:41 459861262
>>459839
>Ведь он же проявляется только вследствие неравномерности распределения массы в теле
Штааа...
Аноним 09/02/19 Суб 01:39:13 459863263
>>459860
Значит в системе есть голономная связь.
Аноним 09/02/19 Суб 18:19:18 459926264
071aekBz6.jpg (145Кб, 660x440)
660x440
270px-NielsBohr.jpg (24Кб, 270x380)
270x380
>>458700 (OP)
В чём конфликт теории относительности и квантовой теории?
09/02/19 Суб 18:46:14 459930265
15497255589001.jpg (24Кб, 270x380)
270x380
hqdefault.jpg (8Кб, 480x360)
480x360
Аноним 09/02/19 Суб 18:58:56 459932266
>>459926
Проквантовать нормально не могут.
Нельзя определить скалярные произведения на гильбертово пространство, линейные операторы нормально не выводятся. В теории возмущений появляются неперенормируемые расходимости.
Может еще чего-то забыл, кто знает попарвит.
Аноним 09/02/19 Суб 23:49:55 459973267
blob (54Кб, 258x195)
258x195
>>459926
Вы задаете вопросы космической глупости
Аноним 10/02/19 Вск 00:01:56 459974268
12.jpg (27Кб, 544x400)
544x400
Аноним 10/02/19 Вск 00:26:02 459975269
blob (256Кб, 600x449)
600x449
>>459974
Два года в наукаче
Аноним 10/02/19 Вск 09:07:49 459986270
>>459932
Что проквантовать?
Аноним 10/02/19 Вск 12:19:33 459991271
kckdaz9zipae-do[...].jpg (63Кб, 600x800)
600x800
Аноним 10/02/19 Вск 12:27:58 459993272
Посоветуйте мне информацию по основным географическим открытиям и исследователям нашей планеты, чтобы прочитать в одном месте, а не рыскать по всем интернетам. Очень нужно, могу арбуз дать взамен.
Аноним 10/02/19 Вск 15:20:16 460005273
Аноним 12/02/19 Втр 15:37:48 460157274
Если звёздных попаданцев закинуть на Андромеду без инфы, то они могут вычислить, что всё ещё находятся на Андромеде, взглянув на небо с современными телескопами?
Аноним 12/02/19 Втр 15:54:33 460160275
>>460157
Тут кто то спрашивал вопрос - если сильно пукнуть на Луне - возможно ли улететь на орбиту.
Это был твой брат?
Аноним 12/02/19 Втр 16:18:47 460163276
>>460160
не, нельзя, слабая тяга
Аноним 12/02/19 Втр 17:13:50 460168277
Беременеют ли гермафродиты? Эксперты с ответов@mail говорят, мешают мужские гормоны, ок, а если их заблочить, чё будет?
Аноним 12/02/19 Втр 17:39:28 460171278
Если бы вместо меня мамкину яйцеклетку оплодотворил бы другой сперматозоид из батиного камшота, сильно ли отличался бы полученный ребенок от меня? Правдивы ли вообще эти мемы, где типа яйцеклетку мог оплодотворить гениальный учёный, но ты успел первым?
Аноним 12/02/19 Втр 17:47:24 460172279
>>460171
>гениальный учёный
Нет таких, хочешь быть таким-сделай так чтобы тебе это было реально интересно и задроть.
Аноним 12/02/19 Втр 17:51:17 460174280
>>460172
Ну хорошо, допустим так: Правдивы ли мемы, где типа яйцеклетку мог оплодотворить сперматозоид, которому была бы интересна наука и который стал бы великим ученым, но ты успел первым?
Аноним 12/02/19 Втр 18:08:05 460177281
>>460174
>интересна наука
>то что должно было передаться генетически
Хех
А знаешь что точно может передаться генетически? Имбицильность.
Аноним 12/02/19 Втр 18:09:25 460178282
>>460177
Иди нахуй короче
Аноним 12/02/19 Втр 22:00:46 460199283
Если центростремительное ускорение направлено к центру окружности-траектории, то почему на тело действует сила, направленная в противоположном направлении?
Аноним 12/02/19 Втр 22:02:07 460200284
>>460199
Потому что она центробежная
Аноним 12/02/19 Втр 22:14:12 460202285
>>460200
Так а чем она обусловлена? Второй закон Ньютона не одобряет
Аноним 12/02/19 Втр 22:20:05 460203286
>>460202
Неинерциальностью системы отсчета.
Аноним 12/02/19 Втр 22:49:25 460207287
>>460199
>центростремительное ускорение направлено к центру окружности-траектории, то почему на тело действует сила, направленная в противоположном направлении?
Только в системе отсчета связанном с телом.
Аноним 13/02/19 Срд 05:25:51 460221288
>>460171
Если у бати не было гениальных генов, то и в тебе им бы неоткуда было взяться. А вообще, ты бы отличался от себя не больше и не меньше, чем от тебя отличается твой родной брат.
Аноним 13/02/19 Срд 08:35:21 460228289
>>460221
>Если у бати не было гениальных генов, то и в тебе им бы неоткуда было взяться.
Мутации хуле.
Аноним 13/02/19 Срд 12:20:43 460244290
15411455385380.jpg (71Кб, 640x640)
640x640
Возраст вселенной 13,8 млрд лет.
При этом радиус наблюдаемой вселенной 46 млрд световых лет.
При этом самый дальний объект во вселенной расположен от нас на расстоянии 13 млрд световых лет.
Почему не дальше?
Если свет до нас не доходит каким-то образом, то почему тогда радиус наблюдаемой 46млрд, а не 13,8млрд?
Или там просто обьектов никаких больше нет?
Почему?
Вики говорит, что диаметр наблюдаемой вселенной 96 млрд световых лет, но в итоге мы не можем видеть дальше, чем 13,8 млрд световых лет?
Аноним 13/02/19 Срд 12:43:11 460248291
800px-Ssc2007-1[...].jpg (184Кб, 800x695)
800x695
Почему, если энтропия увеличивается, то после большого взрыва сформировались звезды, галактики и даже мы?
Аноним 13/02/19 Срд 12:46:32 460249292
>>460244
>При этом самый дальний объект во вселенной расположен от нас на расстоянии 13 млрд световых лет.
Инфу эту поверил?
Аноним 13/02/19 Срд 12:48:34 460250293
>>460248
>сформировались звезды, галактики и даже мы?
Потому что всё это эффективно увеличивает энтропию.
Аноним 13/02/19 Срд 13:14:26 460253294
>>460244
потому что есть еще радиодиапазон, дурилка ты картонная
Аноним 13/02/19 Срд 13:48:45 460255295
>>460244
Ты снова выходишь на связь что ли?
Объяснение тут >>459562
Два предыдущих иксперта сосут хуи.
Аноним 13/02/19 Срд 13:53:32 460257296
>>460248
Ну, всё это лишь временные флуктуации. Энтропия то таки растёт.
Аноним 13/02/19 Срд 14:27:55 460260297
>>460248
>энтропия увеличивается
В изолированной системе.
Аноним 13/02/19 Срд 14:47:01 460262298
>>460260
>>460257
Я не понимаю ваших отговорок. Почему все скукоживается в сложные структуры, если энтропия растет?
Аноним 13/02/19 Срд 14:51:43 460264299
>>460248
А энтропия таки увеличивается.
Впрочем уже на масштабах Галактики термодинамику использовать нельзя. А все из-за того, что слишком большая задержка между двумя областями системы, что автоматом усиливает флуктуации
Ну и второе, Вселенная предпочитает находиться в устойчивом неравновесии - в одном месте увеличиваем в другом уменьшаем.
Аноним 13/02/19 Срд 15:05:14 460267300
>>460262
Потому что пока возникает сложная структура, она срет энтропией только в путь.
Аноним 14/02/19 Чтв 00:10:11 460299301
Не должна ли скорость света накладывать каких-то других ограничений?
Например такого, что не может создаться гравитационное поле, из которого свет не сможет выбраться. А можно только бесконечно близко приближаться к тому, чтобы свет бесконечно долго выбирался оттуда.
Аноним 14/02/19 Чтв 00:32:47 460303302
>>460299
Там ограничения на гладком пространстве. А черная дыра именно что сильное искривление. Математически, больше никаких ограничений нет.
Аноним 14/02/19 Чтв 00:38:57 460305303
>>460303
>Там ограничения на гладком пространстве.
Можешь это подробнее расписать, может я не так это понял.
Аноним 14/02/19 Чтв 01:07:18 460311304
>>460305
Постулируется одинаковость скорости света во всех инерциальных системах отсчета. При этом если мы считаем пространство изотропным и однородным, то быстро приходим к выводу, что одинаковость скорости света дает нам такую характеристику как "интервал" это расстояние в четырехмерном пространстве Минковского (там теорему Пифагора немного поменяли). Дальше можно расширить классическую механику, вывести законы сохранения энергии и импульса. А вот с гравитацией уже сложнее, так как системы неинерциальные. НО! Эйнштейн заметил, что если ты сидишь в ящике, который свободно падает на землю, то для тебя в этом ящике гладкое пространство, там нет гравитации. Так он приходит к выводу, что гравитацию можно описать искривлением пространства Минковского, при этом на бесконечно малых окрестностях можно применить прошлый опыт на гладком пространстве. Так он получил уравнения ОТО. Получается что ограничения скорости света лишь относительно самого пространства, а какие ограничения есть у кривизны никто не знает. Так как никто черные дыры не трогал.
Аноним 14/02/19 Чтв 02:24:58 460318305
Все знают, что чтобы взорвать уран-235 нужны медленные нейтроны. Ну и как, сука, янки ебанули Хиросиму вообще без ёбанного замедлителя- просто тупо сплющив два куска урана в две с половиной критические массы? Я явно что-то не вкуриваю в этой истории.
Аноним 14/02/19 Чтв 07:43:18 460325306
>>460318
>просто тупо сплющив два куска урана в две с половиной критические массы?
Тогда бы ты должен был спросить хули это такое определение у данной массы применительно к урану-235.
Аноним 14/02/19 Чтв 09:18:07 460326307
>>460318
медленные не нужны, потому что
Аноним 14/02/19 Чтв 10:11:42 460327308
>>460325
>>460326
Блин, ну просветите идиота, жалко вам чтоли?
Аноним 14/02/19 Чтв 10:16:36 460328309
Аноним 14/02/19 Чтв 10:47:44 460330310
>>460328
Я понял. Спасибо.
Аноним 14/02/19 Чтв 11:06:09 460332311
15142053924100.png (1268Кб, 1280x853)
1280x853
>>460330
А я не понял. Поясни
Аноним 14/02/19 Чтв 13:11:45 460339312
Это нам на землю столько воды нападало из астеройдов?
Аноним 14/02/19 Чтв 16:38:24 460358313
А что было бы, если бы борионы в теле человека можно было изолировать от поля хиггса?
Аноним 14/02/19 Чтв 16:50:05 460359314
>>460318
>чтобы взорвать уран-235 нужны медленные нейтроны
Нет. Они нужны когда его мало (в процентах от 238-го).
Чистый достаточно хорошо делится и так
>>460328
Аноним 14/02/19 Чтв 16:56:20 460360315
>>460358
Они перестали быть барионами, лол.
Поле хиггса является "частью" электрослабого взаимодействия, ты не можешь одну часть взаимодействия изолировать от другого.
Аноним 14/02/19 Чтв 17:05:05 460362316
>>459500
Не выдержал блин, нихуясебе заявление.
Человек не может сожрать большую кошку? Пятеро смогут!
Человек замерзнет из-за того что голый? Кошкина шкура поможет согрется!
Еще сюда про выносливость и способность выживать после совершенно жутких ранений, если мозг и сердце в рабочем состоянии.
Аноним 14/02/19 Чтв 17:10:55 460363317
>>460358
Изолировать не сможешь, но можешь восстановить нарушенную симметрию с повышением энергии этого самого поля.
Аноним 14/02/19 Чтв 17:15:09 460364318
>>460339
Астероиды прилетают к нам почти без воды. Более того Земля теряет свою воду.
Дело в том, что с Солнца "дует" солнечный ветер - поток заряженных и не очень частиц. Эти частицы стакливаясь с молекулами воды вызывают ее распад на OH и атом/ион водорода. Поскольку водород/протон достаточно легкие, они получают существенную скорость, причем достаточную чтобы вылетит и навсегда покинуть свое место.
Даже для больших тел как Земля или Венера, гравитационного поля не достаточно чтоб удержать водород, поэтому он со временен улетучивается, а звездный ветер еще вытаскивает водород из воды/метана/аммиака.
В прочем кометы в основном состоят из льда, и если их не успеет испарить Солнце, то вполне могут принести немного воды на планету. Но максимум мы имеем максимум тонну за раз и это довольно редкое события. Более мелкие тела уже теряют водород и к нам прилетает кусок углекислого газа, сажи и иногда всякой азотистой органики.
Аноним 14/02/19 Чтв 17:18:33 460365319
>>460363
Ой, то есть повышением энергии других квантовых поле.
Аноним 14/02/19 Чтв 17:20:22 460366320
sky-line.jpg (78Кб, 1024x596)
1024x596
Я застану эту хуйню при жизни, если мне 20 лет?
Аноним 14/02/19 Чтв 17:21:26 460367321
>>460366
В такой виде - никогда
Аноним 14/02/19 Чтв 17:37:57 460369322
>>460364
Я Просто не понимаю как на горящем куске камня появилось столько воды, чтобы началась реакция? Облака и все такое?
Аноним 14/02/19 Чтв 18:33:53 460372323
>>460367
Я имею в виду концепцию
Аноним 14/02/19 Чтв 19:47:48 460377324
>>460372
Может быть, кто знает.
Аноним 14/02/19 Чтв 22:54:40 460397325
>>460369
На Солнце тоже вода есть.
Аноним 14/02/19 Чтв 23:31:12 460404326
sdgsdaasc.jpg (108Кб, 831x813)
831x813
Что за микроскоп? Модель?
Аноним 15/02/19 Птн 12:16:34 460418327
>>460404
МБИ-15
Микроскоп биологический исследовательский МБИ-15
Аноним 15/02/19 Птн 13:46:28 460422328
>>460339
да, в виде гидроксидов, и сейчас в мантии на порядок больше воды чем в океане
Аноним 16/02/19 Суб 13:21:41 460488329
Если влить перегретую воду в переохлаждённую (и наоборот), чё будет? Лёд в бурлящем взрыве/одно из них/нихуя?
Аноним 16/02/19 Суб 19:46:23 460513330
Объясните за мощность двигателей в авиации. Ну допустим понятно, что обороты и винт влияют на то, как будет тянуть связка двигатель + винт. Но на что влияет мощность двигателя в таком случае?

Допустим есть винт диаметром метр и массой 5кг. Раскрутить его сможет одинаково успешно до допустим 2000 оборотов и двигатель с мощностью 10квт, так и двигатель с мощностью 50квт. В чем преимущество последнего и зачем нужна вся эта мощность?
Аноним 16/02/19 Суб 20:50:16 460515331
IMG201902162047[...].jpg (106Кб, 720x547)
720x547
>>460513
Ну удачно тебе полетать с таким винтом.
Аноним 16/02/19 Суб 21:10:43 460516332
>>460513
Тебе же не просто винт раскрутить - тебе массу самолета тянуть надо
Аноним 16/02/19 Суб 21:11:15 460517333
>>460513
Тебе же не просто винт раскрутить - тебе массу самолета тянуть надо
Аноним 16/02/19 Суб 21:22:16 460518334
2b0457b313c1dc0[...].jpg (46Кб, 500x713)
500x713
Аноны, как думаете сильно ли тупеешь от игр?
Уже пожилым стал, а привычка играть так и осталась.
Вот у вас тут у большинство 150+ iq, вы в какие-нибудь игры в свободное время играете?
(Имеются ввиду компьютерные игры)
Аноним 16/02/19 Суб 21:58:26 460519335
>>460518
>тупеешь
Сначала определись со своим маняопределением этого.
>iq
Если определять умение решать тестики решением тестиков, то игры скорее всего да, понизят iq, хотя это зависит от того, насколько схожи задачи и действия в играх будут к задачам и действиям в тестиках, могут и увеличить iq.
Аноним 16/02/19 Суб 22:08:03 460520336
>>460519
По другому вопрос задам. Играют ли аноны, научных и около научных профессий в игоры?
Аноним 16/02/19 Суб 22:18:49 460521337
>>460520
Поигрываю иногда в старую годноту. От онлайновых и мультиплеерных ушел после окончания вуза, с тех пор только сингл.
Аноним 16/02/19 Суб 22:24:18 460522338
>>460521
А сколько примерно в неделю (в часах) тратишь?
Аноним 16/02/19 Суб 22:28:48 460523339
>>460518
В основном всякие рогалики, парадоксодрочильни, всякие аутизм типа sunless sea, и.т.д У ааа йобы только тошнит, ибо геймплея в ней, как правило нуль.
Аноним 16/02/19 Суб 22:36:28 460524340
>>460516
>>460517
Ну и в чем проблема, если винт подходит под планер по характеристикам? Вот стоишь ты на земле на тормозах и раскручиваешь винт до 2000 оборотов с маломощным двигателем. Он спокойно раскручивается. И тут ты снимаешь тормоза и что винт внезапно перестаёт работать? Что за магия?

>>460515
Спасибо за ответ на вопрос в треде тупых вопросов (нет, иди нахуй).
Аноним 16/02/19 Суб 22:57:20 460525341
>>460524
>Спасибо за ответ на вопрос в треде тупых вопросов
Ты слишком тупой для треда тупых вопросов, попробуй /ussr
Аноним 16/02/19 Суб 23:45:16 460527342
>При контакте материи с антиматерией она аннигилирует с полным переходом массы в энергию по е=mc^2.
>В вакууме постоянно возникают пары частица-античастица которые практически сразу аннигилирует, поэтому кажется что там ничего не происходит и пусто.
Эта аннигиляции не аннигиляция, и вообще она уволилась из аннигиляций месяц назад?
Аноним 17/02/19 Вск 00:35:29 460530343
>>460518
От самих игр тупить не должен, по идее.
Проблема только если эти игры настолько тебя занимают, что ты ни на чём другом уже сосредоточиться не можешь
Аноним 17/02/19 Вск 09:05:22 460546344
1.Если бы породы собак были естественного происхождения, они бы причислялись к разным видам? Или они всё ещё практически идентичны генетически?
2.Если в геноме зародыша поменять единственную пару оснований (на значимом участке), он будет отбракован/родится с дефектами/похуй, геном защищён от мелких багов?
Аноним 17/02/19 Вск 09:53:32 460547345
>>460530
Тут немного сложно. Обычно после работы есть часа два свободных, которые можно потратить на прокачивание навыков, языков ну и пр.
Обычно в это время деградирую просмотром ютуба, сериальчиков и игрулями.
Заниматься умственной деятельностью (изучением, обучением, запоминанием) довольно тяжело после раб. дня.
Тут собственно вопрос - это у меня мозг не приучен к такой нагрузке и можно решить это ограничив свои "хобби" и через силу заниматься обучением. Может через пару месяцев будет получше.
Или это сказывается простая усталость и таким темпом просто загоню себя и профита будет мало.
Аноним 17/02/19 Вск 11:00:45 460549346
>>460547
>после работы
>часа два свободных
>можно потратить
>можно
С этого и надо было начинать, а не с игр, тебя уже не спасти, ты конченный имбицил. Падшее тело, опустившееся.
Аноним 17/02/19 Вск 11:02:12 460550347
>>460549
Тебя смущает сам факт работы? Не совсем понял.
Аноним 17/02/19 Вск 11:18:16 460551348
>>460550
Меня ничего не смущает. А весь твой набор-аховый. Отупеть от видеоигр-это последнее о чём тебе стоит беспокоиться.
Аноним 17/02/19 Вск 11:32:18 460555349
>>460551
А о чем тогда стоит?
Я все еще не понял твою мысль.
Аноним 17/02/19 Вск 13:20:56 460556350
15494823901640.mp4 (18393Кб, 640x360, 00:04:48)
640x360
Смотрите в самом конце, как рыба додумалась снабжать икру кислородом
Аноним 17/02/19 Вск 15:33:38 460560351
>>460546
Главный критерий вида это способность создания плодовитого потомства. Собаки разных пород очень даже хорошо скрещиваются.
Аноним 17/02/19 Вск 20:10:17 460572352
Сильное и слабое взаимодействия это тоже поля?
Аноним 17/02/19 Вск 20:33:09 460573353
>>460572
Да, эти взаимодействия рассматриваются через поля, да еще калибровочные.
Аноним 17/02/19 Вск 20:36:28 460574354
>>460573
Разве напряжённость не всех полей убывает/должна убывать по закону обратных квадратов?
Аноним 17/02/19 Вск 20:58:38 460576355
>>460574
Нет. Причем тут напряженность? Или ты представляешь поле как некие силы действующие на пробный заряд/частицу?
От этого формализма уже отошли после того как сформировали концепцию физического поля.
Аноним 17/02/19 Вск 21:33:39 460578356
>>460576
>напряженность
Наверное лучше было сказать сила воздействия.
>концепцию физического поля.
Понял, почитаю.
Аноним 17/02/19 Вск 21:54:18 460579357
Аноним 18/02/19 Пнд 02:00:41 460593358
Аноним 18/02/19 Пнд 02:22:14 460594359
>>460593
Процессы называют одинаково, и описывают одинаково, но в первом случае выделяется огромное количество энергии, вся масса переходит в энергию, а во втором "исчезают без следа" и всё проходит незаметно, что за херня?
Аноним 18/02/19 Пнд 03:08:34 460597360
>>460594
>что за херня?
Как изначально ничего нет, так и в конце ничего нет. Это немного сложно объяснить.
Аноним 18/02/19 Пнд 09:59:01 460612361
>>460597
>Как изначально ничего нет, так и в конце ничего нет.
Ты не так понял мои посты и имеешь ввиду то что "тяжело объяснить очевидную вещь", или ты о том что этот процесс реально сложно объяснить?
Аноним 18/02/19 Пнд 11:01:15 460614362
Стикер (127Кб, 512x382)
512x382
>>460612
>очевидную вещь
Не очевидно там. Дело в том, что в квантовой механике нет понятий траекторий и т.п. Там берут амплитуду вероятности и складывают по всем возможным ситуациям. Если бы ты был квантовой частицей, и пошел бы на работу, то я заметив тебя на работе, не просто не мог бы сказать как именно ты приехал на работу, а абсолютно точно сказал бы, что ты приехал всеми возможными способами. У каждого твоего пути, была бы своя комплексная амплитуда, которая сложилась бы в итоговую амплитуду вероятности, что ты пришел на работу. Вся мякотка в том, что помимо довольно очевидных "ты поехал на машине" и "ты поехал на автобусе". Есть и не очень очевидные, которые стоит учитывать "ты пошел и угнал машину, ограбил банк и приехал на работу" или "ты выебал сам себя в задницу, родил свою копию, убил себя изначального и на работу пришла твоя копия". Вот как раз с последним вариантом и работают, когда говорят про аннигиляцию в вакууме.
Аноним 18/02/19 Пнд 13:00:02 460623363
>>460614
Вдумчиво перечитал и всё ещё не уверен что речь именно о том что нужно.
Античастица рождающаяся в такой паре не равно обычной античастице или антиматерии?

Олсо
>Дело в том, что в квантовой механике нет понятий траекторий и т.п...
Это всё реально так, или это всё из-за невозможности измерить не повлияв/абстракция из-за невозможности измерить не повлияв?
И если это реально так, то какие доказательства этого?
Аноним 18/02/19 Пнд 13:43:33 460637364
>>460623
>Это всё реально так
this

ЭПР парадокс например. Еще всякие квантовые эксперименты с частицами, по типу "квантового ластика", "с отложенным выбором" или самый ебнутый как по мне это "квантовая бомба". Взрывает мозг.

Аноним 18/02/19 Пнд 13:52:07 460639365
>>460637
>эксперименты
Эксперименты не подразумевающие измерения?
Аноним 18/02/19 Пнд 14:00:28 460640366
>>460639
Ну например измерение "после" тебе подойдет? Или "измерение с уничтожением результата измерения". Говорю же, посмотри что это за эксперименты, там дикости для "здравого смысла".
Аноним 18/02/19 Пнд 14:07:02 460643367
>>460623
Другой анон. Переформулирую по другому.
В квантовой теории нет никаких частиц, вообще. Вся материя существуют в своем уютном бесконечномерном пространстве манямирке, где почти все возможно. Однако этот манямирок обладает определенным свойствами как гладкость, линейность и всяческие симметрии, это свойства очень важные, они и определяют саму реальность.
Далее есть такие вещь как операторы.. Они заставляют трещать манямирок так, чтобы только самые альфачи из пространства манямирка оставались. И эти альфачей есть определённые характеристики, как рост, вес, размер хуя. И вот "частичность" тоже является характеристикой оператора. Операторы могут забавным образом накладываться. Поэтому прикладывая почти любой оператор, мы автоматом получаем все возможные вариантов частиц, и это называется рождением-уничтожением частиц. И в зависимости от операторов, которые так же определяют граничные условия, а мы получаем какие частицы и с какими свойствами нам будут возможно видны.
Но оператор это идеальный случай, на практике возникает целых ворох наложений, создающие дополнительные возмущения и нелинейности.

Отдельно об изменение в квантовой теории. Любое измерение это воздействие сложной системы на простую "элементарную" систему, из разности сложностей возникает неполнота измеряемых величин. Ситуацию усугубляет то, что для измерений нам усреднять состояния сложной системы(детектора), из-за это возникает принципиальная невозможность непосредственно смотреть на квантовые состояния "элементарный" систем.
Аноним 18/02/19 Пнд 14:28:09 460649368
>>460594
Сам не особо въезжаю пока в эти квантовые флуктуации, но вроде бы к виртуальным частицам не применимы законы сохранения в классическом понимании. Точнее там их масса не связана с энергией и импульсом. Можешь сам подумать, если у тебя некоторая частица испустит еще виртуальную, ее должно отбросить назад. И для этого всего нужна дополнительная энергия, которой "внутри" исходной частицы быть не может. Так же, видимо, и при их аннигиляции они способны аннигилировать в ничто, в отличии от реальных частиц.
Аноним 18/02/19 Пнд 14:34:11 460651369
>>460649
>с энергией и импульсом.
Просто никто не запрещает отрицательные энергии и комплексные импульсы.
Аноним 18/02/19 Пнд 14:51:23 460659370
>>460651
Возможно и так, но не
>в классическом понимании
Аноним 18/02/19 Пнд 14:58:46 460662371
изображение.png (446Кб, 452x552)
452x552
Сейчас читал статью на вики про фетбайки и там указано что
>При эксплуатации давление в камерах находится в пределах от 0,2 атм. до 2 атм.
Что за фигня подумал я? Давление 0,2 атмосферы? Это как? Зашел нас страницу футбольного мяча
>имеет давление, равное 0,6—1,1 атмосферы

Как это понимать? Как внутреннее давление может быть ниже давления среды и при этом оно не скукожится нахуй?
Аноним 18/02/19 Пнд 15:06:04 460667372
>>460662
Давление в камерах/мечах это разница давленией между средой и камерой.
Аноним 18/02/19 Пнд 15:06:45 460668373
>>460662
Измеряют относительно одной уже имеющейся.
Если хочешь узнать давление относительно вакуума-прибалвляешь +1.
Аноним 18/02/19 Пнд 16:48:33 460679374
Какой тип вычислительной машины В ТЕОРИИ самый пиздатый по всем фронтам, в который и свою личность загрузить не жалко? Что лучше топового мозга?
Аноним 18/02/19 Пнд 16:50:44 460680375
>>458700 (OP)
Расскажите о ленте Мёбиуса, будто рассказываете гуманитарию. Любые интересные (и не очень) фичи.
Аноним 18/02/19 Пнд 16:52:01 460683376
>>460679
Кластер специализированных ИИ.
Аноним 18/02/19 Пнд 16:56:11 460684377
>>460680
Единственное интересное свойство, что поверхность неориентируемая и забавное считаются геодезические.
Аноним 18/02/19 Пнд 18:21:56 460689378
>>460684
>забавное считаются геодезические
Как?
Аноним 19/02/19 Втр 02:16:33 460714379
У меня плохо с английским, скажите, существует ли автоматический способ определить сверхпроводимость материала?
Аноним 19/02/19 Втр 02:19:50 460715380
>>460714
З.Ы. про английский это я имею ввиду, что не смог качественно загуглить свой вопрос
Аноним 19/02/19 Втр 02:56:28 460717381
>>460714
Ты имеешь ввиду рассчитать температуру перехода и тип сверхпроводимости?
Аноним 19/02/19 Втр 03:12:18 460719382
>>460717
Ну, я бы хотел узнать, существует ли какая-нибудь автоматическая штука в которую можно закинуть материал, и чтобы эта штука постепенно понижала температуру и проверяла стали материал сверхпроводимым. Или может быть какую-нибудь магию бы с магнитыми полями делал. Чтобы по итогу выдать результат - данный материал не может стать сверхпроводником, пытались шаманить и ничего не получилось.

Это для шараги нужно, я нашел в интернете статью где статистически вычисляют критическую температуру при которой материал мог бы стать сверхпроводником, но модель может ошибиться и вообще модель не дает гарантии, что материал вообще станет сверхповодимым при предсказываемой температуре.

В шараге я мне надо будет словесно вступиться базаром за модель из той статьи и сказать, что существуют и другие способы лабораторного перебора и поиска температуры, определения является ли материал сверхпроводимым и что ошибки не страшны, ибо их можно выловить автоматически другим способом без моделирования и статистики.
Аноним 19/02/19 Втр 03:26:30 460721383
>>460714
>>460717
А хотя не надо, думаю, не буду сверхпроводимость брать, наверное, заклюють меня, что слишком дорого перебор в лабе делать, что ошибка слишком дорога. Лучше другие статистические модели пошарю, где ошибка хуйня собачья.
Аноним 19/02/19 Втр 08:37:41 460731384
По телеку услышал про микропластиковые волокна, мол, они при каждой стирке вымываются из синтетичской одежды и попадают в воду, а дальше, конечо, в живые организмы в т.ч. людей. Это типа очень плохо и является экологической проблемой. Отфильтровать их сложно в виду крайне малого размера. Вопрос а какой вред они наносят если они не разлагаются и не вступают в хим. реакции в теле(пластик же) ? Т. е. они могут только механически навредить, но ввиду малого размера наверно и тромб не смогут образовать, не ?
Аноним 19/02/19 Втр 15:22:15 460758385
А в обычном железном магните практически все атомы одинаково магнитно ориентированы или всё-таки ещё дофига рандомных? А то слабоватое поле получается: обычно же когда какой-то параметр одинаков на всех уровнях, его значение огромно
Аноним 19/02/19 Втр 23:12:08 460784386
15274244130552.jpg (87Кб, 822x720)
822x720
Медач слишком быдляч, в психаче тупые дети с фелософской кашей в голове. Анон, ты ведь строение и функционирование мозга изучал? Я с недавнего времени говорю не то, что думаю. Не оговорки по фрейду, ничего такого. Просто вот нужно сказать "1965", я вижу в голове эту цифру или смотрю на неё написанную, но я всё равно могу произнести "1985", к примеру. Что за блядь, ну что.., Эх. Хочу сказать "да", говорю – "нет" и так даже с целыми фразами, ну ёбана.
Аноним 20/02/19 Срд 00:04:50 460786387
>>460784
Лучше вставляй свечу беги к врачу. Вдруг какой микроинсульт, так можно постепенно и овощем стать.
Аноним 20/02/19 Срд 00:26:52 460788388
>>460786
Если и был, то ладно, только деньги выброшу на МРТ.
Аноним 20/02/19 Срд 01:00:09 460789389
>>458700 (OP)
У Хокинга есть краткая история времени, есть кратчайшая, есть еще какая-то Крупномасштабная структура пространства-времени, про будущее еще и про край вселенной. Как правильно читать? Кратчайшую до краткой или наоборот? И почему?
Аноним 20/02/19 Срд 01:03:20 460790390
>>460789
Слишком тупой вопрос даже для этого треда. Читай как хочешь. Вряд ли они связанны в серию. Можешь посмотреть по датам издания.
Аноним 20/02/19 Срд 01:17:23 460791391
3eb96ef8fa434b8[...].jpg (47Кб, 602x481)
602x481
CHECK CHECK
DUBSALOO
Аноним 20/02/19 Срд 01:24:57 460793392
>>460789
Еще и мир в скорлупке, блядь.
>>460790
Они связаны идейно, нет?
https://ru.wikipedia.org/wiki/%D0%A5%D0%BE%D0%BA%D0%B8%D0%BD%D0%B3,_%D0%A1%D1%82%D0%B8%D0%B2%D0%B5%D0%BD#%D0%91%D0%B8%D0%B1%D0%BB%D0%B8%D0%BE%D0%B3%D1%80%D0%B0%D1%84%D0%B8%D1%8F
Хер там чего по годам поймешь, особенно когда куча изданий, а у нас они переводились в непонятной последовательности.
Аноним 20/02/19 Срд 03:13:56 460799393
>>460793
идейно это всё сильно связанно. просто графомания. предельно просто и красивыми метафорами. Если интересно, читай что-нить поновоее с названием, которое тебе больше по душе. Я не читал и не уверен, что тут такие люди найдутся.
Аноним 20/02/19 Срд 03:30:24 460801394
>>460799
Почему? Это же классика научпопа про космос.
Аноним 20/02/19 Срд 03:54:12 460802395
>>460801
Как по мне лучше послушать нормальные лекции, просто опуская матан, если совсем голову забивать не охота. К тому же есть и научно-популярные лекции неплохие, где всё же научного больше, чем популярного. Старые книги хоккинга же в целом уже достаточно устарели. Новые судя по всему в большей мере философский бред об устройстве вселенной.
Аноним 20/02/19 Срд 05:10:40 460805396
Аноним 20/02/19 Срд 05:11:24 460806397
Аноним 20/02/19 Срд 06:04:11 460807398
>>460805
Хуита.
Начиная с квантовой теории уже работают с бесконечномерными пространствами. Ну или хотя бы в классической физике, симметрии поля не вкладываются в 3-х мерное пространство.
Для этого используют определенные методы, позволяющие работать с такими объектами.
Аноним 20/02/19 Срд 08:46:14 460811399
image.png (296Кб, 900x507)
900x507
а что, если сфотографировать атом на очень длинной выдержке?
Аноним 20/02/19 Срд 10:22:10 460819400
Снимок.PNG (913Кб, 787x443)
787x443
>>460811
Получится пикрелейтед
Аноним 20/02/19 Срд 10:57:06 460821401
>>460819
а электронное облако, оно будет как на моём пикриле, или..?
Аноним 20/02/19 Срд 19:37:29 460843402
Если кто-то гениальный понял, как создать идеальное топливо, то он создатель может открыть фабрику и создавать топливо монопольно (скрыв секрет создания топлива)? Ну то есть, что происходит с открытиями, которые способны изменить мир? Могут ли их создатели скрыть свои секреты и торговать плодами открытия, не рассказав, как они это сделали? (Блин, сори, трудно сформулировать) Ну например, я создам новый метод реанимации людей, могу ли я открыть клиники, единственное место с такой возможностью? Ну типа я не скажу, как "воскрешаю" люедй. Или государства всего мира схватят меня за яйца и заставят открыть миру принцип работы, чтобы всякие бесталанные дельцы тоже могли делать деньги, хотя они ничего не сделали по сути, тупо нахлебники...
Аноним 20/02/19 Срд 19:42:58 460844403
>>458706
>одно только еврейство неплохо кормит.
Что-то меня не кормит.
Мимоеврейпомаме
Аноним 20/02/19 Срд 19:49:40 460847404
>>460843
>Ну то есть, что происходит с открытиями, которые способны изменить мир?
Такие открытия только в комиксах происходят. В мире же монополия на технологии держится на патентах, которые подкрепляются авианосцами. Захочешь нагло копировать и сам продавать и тебя быстро демократизируют.
Аноним 20/02/19 Срд 19:51:11 460848405
>>460847
Ну а если появится настолько крутая технология, что изменяет мир, то все начнут её воспроизводить, плевав на санкции, патенты и даже авианосцы. Никому ведь патенты на ЯО и АЭС интересны не были.
Аноним 20/02/19 Срд 19:54:54 460850406
>>458877
Ну например Алису. Только не которую мы заслужили, а которая вспомнит о чем я с ней говорил на прошлой неделе и разовьет мысль. Которая будет с моих произвольно сформулированных команд управлять телефоном как я управляю им пальцем.
Аноним 20/02/19 Срд 19:56:24 460851407
>>458885
>Если соблюдаются определенные условия, человек ржет.
А его сосед не ржет.
20/02/19 Срд 20:27:02 460853408
>>460850
Управляю тобой пальцем.
На колени, животное.
Аноним 21/02/19 Чтв 14:48:44 460908409
>>458872
Давно уже сделали, на гите куча репозиториев ML, во всяких Open AI тысячи волонтеров трудятся
Аноним 21/02/19 Чтв 18:49:58 460920410
Можно ли вызвать грозу зимой? Если да, то каким образом? Под зимой я подразумеваю много снега и холод.
Аноним 21/02/19 Чтв 20:03:13 460927411
Если я перевожу взгяд с пальца перед лицом на какую-нибудь туманность андромеды, то мой взгляд от пальца до туманности достигает в миллионы раз быстрее скорости света.
Вот так есл взять провести от пальца вектор до центра туманности и повесить на неё ползунок, который управляется моим взглядом, то ползунок за долю секунды пролетает несколько световых лет.
Эта мысль не даёт мне покоя.
Аноним 21/02/19 Чтв 20:55:11 460933412
>>460927
Потому что идея не имеет пространственного воплощения. А может быть, потому, что твоя сетчатка двумерная и ты, физически говоря, переводишь взгляд с одного предмета с Z=0 на другой предмет с Z=0. Как тебе приятнее думать.
Аноним 21/02/19 Чтв 21:08:52 460935413
>>460927
Это как солнечный зайчик, который может двигаться быстрее скорости света. Но не может переносить информацию.
Аноним 21/02/19 Чтв 21:29:05 460937414
Почему для человека температура воздуха и воды 36.6 градусов это жарко?
Аноним 21/02/19 Чтв 21:46:53 460938415
Есть какие-то ассоциативные обоснования, почему кварки нарекли очаровательными, прелестными и тд, или это от балды взято?
Аноним 21/02/19 Чтв 22:34:59 460943416
>>460937
Потому что температура кожи 32 градуса. Это в кишках у тебя 36.6, а снаружи у тебя радиатор для охлада.
Аноним 21/02/19 Чтв 22:43:14 460945417
>>460943
Но и 30 жарко пиздец, так что аргумент инвалид.
Аноним 21/02/19 Чтв 23:01:48 460947418
>>460945
Он имеет в виду, что организм устроен так, чтобы постоянно рассеивать тепло в окружающую среду.
Когда температура внешней среды становится около 30, тепловой поток сильно снижается, организм уже не может рассеивать тепло и начинает перегреваться, что для мозга интерпретируется как "жарко".
Аноним 21/02/19 Чтв 23:13:48 460949419
>>460938
Да, от балды.
Кваркам было множество названий, закрепились потому что МЕМАСИК нахуй.
Верхний и нижний кварк пошли от проекции изоспина.
Странный пошли от "странности", когда открывали зоопарк адронов, выделили ебанутые частицы, который приписали характеристику "странность". Остальные придумали по такой же аналогии заранее, когда пилили теорию.
Аноним 22/02/19 Птн 00:47:03 460954420
Присваиваются ли частицам и объектам стандартной модели и макрофизики какие-либо координаты свёрнутых измерений в теориях струн? Например, есть мячик, в наших трёх измеренияъ ему присваиваются три множества координат, в остальных 6 измеринях тоже?
Аноним 22/02/19 Птн 00:59:21 460955421
>>460954
>в наших трёх измеренияъ
У нас не три размерности, а шесть. Три было бы, если бы вселенная была статичной.
Аноним 22/02/19 Птн 01:04:02 460958422
Сап, зачем профессиональные дрифтеры ставят на свои машины антикрылья? Как антикрыло будет работать если машина преимущественно едет боком? Не будет ли он добавлять лишнего держака?
Аноним 22/02/19 Птн 13:40:20 461002423
>>460958
Повадки и приметы автоживотных это вне компетенции этой борды, тебе в автач.
Электродинамика Аноним 22/02/19 Птн 18:10:35 461009424
Подскажите общую более формулу Томсона, как расчитывать период, когда у тебя больше одного конденсатора/катушки?
Аноним 22/02/19 Птн 19:49:24 461013425
>>461009
Комплексные числа знаешь?
Аноним 22/02/19 Птн 19:50:15 461014426
Аноним 22/02/19 Птн 19:51:35 461015427
Аноним 22/02/19 Птн 20:52:01 461016428
атом всё.JPG (85Кб, 1270x673)
1270x673
Ну че уебища чернобыльские, как сосется?
Аноним 22/02/19 Птн 22:28:39 461017429
>>461016
Партия зеленых запрещает в Германии АЭС, так как решает бороться за экологию с помощью сжигания угля.
Аноним 23/02/19 Суб 00:51:39 461018430
>>461017
> Партия зеленых
А ты не выдумываешь? Разве зеленые петухи хоть в какой-то стране имеют реальный вес и места?
Аноним 23/02/19 Суб 04:42:18 461019431
Аноним 23/02/19 Суб 14:09:16 461034432
>>461018
Почему зелёные это не главенствующая политическая сила в мире? Разве не очевидно, что защита природы это наша первоочерёдная обязанность, какие бы идеологические и экономические терки в мире ни существовали.
Аноним 23/02/19 Суб 14:23:36 461035433
>>461034
>Разве не очевидно, что защита природы это наша первоочерёдная обязанность
Потому что они тупые долбоебы. Гринпис на зарплате нефтедолларов сидит. А партия зеленых в Германии на полном серьезе запрещают АЭС.
Аноним 23/02/19 Суб 17:17:03 461044434
>>461035
С тем, что существующие зелёные долбоёбы, никто и не спорит. Я говорю о самой зелёной идеологии.
Аноним 23/02/19 Суб 17:52:29 461048435
>>461044
>Я говорю о самой зелёной идеологии.
Удачи запрещать извержения вулканов.
Аноним 23/02/19 Суб 18:41:31 461050436
>>461035
>А партия зеленых в Германии на полном серьезе запрещают АЭС.
Их нужно в Чернобыль на экскурсию свозить, там такая природа сейчас - закачаешься. Рыбы с лодку размером. Зачем запрещать? Пусть взрываются.
Аноним 23/02/19 Суб 18:56:40 461051437
Как посчитать за какое время испарится шар из чистого железа в вакууме, при нормальной температуре, диаметром ,скажем, в 1 метр?
Аноним 23/02/19 Суб 20:16:27 461054438
А может быть такое что расширение пространства действует только на свет растягивая только его, и не влияет ни на что больше, и поэтому нам только кажется что галактики вокруг разлетаются, а это просто свет растянут? Ведь даже нейтрино проходят сквозь Землю, почему пространство должно расталкивать галактики, а не проходить сквозь?
Если можно ответить подробнее, а то физика у меня только хобби.
Аноним 23/02/19 Суб 21:38:32 461062439
>>461054
Пространство это умозрительная манякатегория. Расширение это умозрительная манякатегория. Мы 2.5-мерные существа в дохуямерной Вселенной. Мы фантазируем как умеем.
Аноним 23/02/19 Суб 21:55:51 461063440
>>461054
>может быть такое что расширение пространства действует только на свет растягивая только его
Тогда тебе придется половину физики выкинуть на помойку. Особенно в той части, где у тебя материя превращается в свет. (аннигиляция например)
Аноним 24/02/19 Вск 01:15:20 461074441
15207838379803.jpg (38Кб, 596x464)
596x464
Какие гормоны заставляют женский организм обделять мозги питанием и посылать ресурсы на нужды деторождения? Да и вообще, какие гормоны в женском теле бьют по мозгу больше всего. Я знаю только то, что тесто делает психику увереннее и кортизол при нормальном его уровне не так ебошит по тканям как без него, да и вообще всё, что делает пофигистом сохраняет мозги. А что там по разнице наличия стимуляторов работы и росту нейронов между м и ж?
Аноним 24/02/19 Вск 12:51:43 461087442
>>460935
Откуда вообще есть пошел этот высер про солнечный зайчик быстрее скорости света? КАК блядь то что состоит из света может двигаться быстрее света. Ну ебанутые
Аноним 24/02/19 Вск 13:01:39 461088443
>>458872
Ты исходишь из (неверной) предпосылки что надо просто кинуть побольше народа на проблему и она обязательно будет решена. На деле для прорыва нужна как минимум одна гениальная личность вроде Эйнштейна. Или можно посмотреть на Перельмана или Уайлса, как они двадцать лет бились над проблемой. А теперь переведем взгляд на современно комьюнити сеточников почти целиком состоящее из хипстушков-хайпожоров, которые сегодня вкатываются в сеточки и засирают интернет еще одним туториалом о том как вкатываться в сеточки. А завтра в моде буду опердени и они так же переберутся на написание туториалов "об оперденях для самых тупых за один день без матана".
Аноним 24/02/19 Вск 13:38:51 461090444
>>461087
Ну включи голову
Аноним 24/02/19 Вск 13:47:10 461091445
>>461090
Сам попробуй, клоун
Аноним 24/02/19 Вск 14:26:24 461099446
f74bcd.png (121Кб, 300x300)
300x300
>>461062
Так можно очень далеко уйти, с таким подходом.
>>461063
Ну конечно не только на свет это должно действовать, но на "материю" намного меньше, галактики немного разлетаются, но не с ускорением, а ускорение получается от того что свет растягивается. Ещё недавно проскакивала тема, что у Вселенной возможен двойник, где время идёт в обратную сторону, но ведь это две части одной Вселенной получается, значит там просто процессы в другую сторону идут, и тут возникает вопрос был ли БВ или Вселенной в несколько раз больше лет и за всё это время сформировалась такая мега-"ЧД", которая с одного места всё затягивает и сжимает, а с другого выталкивает и расширяет, и ещё крутится в разные стороны, может такое быть?
Аноним 24/02/19 Вск 15:10:33 461100447
>>461099
> такая мега-"ЧД"
Вполне себе
>может такое быть?
CPT-симметрия на это намекает
Аноним 24/02/19 Вск 15:45:11 461102448
>>458700 (OP)
вопрос не тупой (наверное), но не стоит отдельного треда
никто не подскажет методы выявления плазмид у бактерий?
психология методика Аноним 24/02/19 Вск 15:50:19 461103449
Ребята, если кто-нибудь зашарит за то как обработать эту методику :
https://sites.google.com/site/test300m/sro
и сможет объяснить - отпишитесь для связи, пожалуйста, помогите тупице.
Аноним 24/02/19 Вск 18:39:53 461105450
>>461100
Мне это казалось каким-то неисчерпывающим, поэтому немного сказочным. Надо физику подучить.
Аноним 24/02/19 Вск 19:07:09 461106451
>>461103
В смысле "обработать"?
Аноним 24/02/19 Вск 19:33:51 461107452
вредно ли есть мумифицированный труп?
Аноним 24/02/19 Вск 22:44:21 461111453
polefiltr9-430x[...].jpg (55Кб, 430x347)
430x347
222.png (154Кб, 1141x751)
1141x751
155086111217068[...].jpg (79Кб, 700x525)
700x525
Безымянный.png (43Кб, 1284x743)
1284x743
Посмотрев как украинцы у себя на форумах греют бассейн тупо кидая шланг с дырочками на крышу и собирая воду с раскаленного металла (30 квтч с 80 метров(много испаряется, 0.5 куба в неделю, нужна пленка и герметизация по коньку крыши) и как Канадцы отгрохали эко послелок в котором все лето куча коллекторов греет утепленное поле земли и потом всю зиму с него снимает тепло
МЕНЯ ОСЕНИЛО, ЭВРИКА!

1) строим домик
2) строим односкатную крышу
3) кладем листовую металлическую кровлю черную, желательно матовую
4) проклеиваем желоба скотчем, что бы защитить металл от воды
5) а сверху для повышения КПД дешевую пленку от теплицы и тонкий слой пенопласта с обратной стороны ещё можно
6) ТУПО КАЧАЕМ ВОДУ СО СКВАЖИНЫ НА КРЫШУ И СБРАСЫВАЕМ В ДРЕНАЖ ПОД ДОМ!

Допустим, дом 6 на 10 метров, крыша 30 градусов, около 70 метров. КПД с пленкой и черным цветом будет гораздо выше вместо того, что получили украинцы, около 70%. 49Квт
ч на 6-7 часов в день = 300квт*ч в сутки, допустим 1 градус одной тонны воды дает 1квт в час, то есть с 20 до 70 градусов, это 50Квт с тонны, нужно всего лишь прокачать 6 тонн за 6 часов и что бы за сутки они успевали впитываться.

Мы копаем трактором под место будущего фундамента 10 траншей по 10 метров с расстоянием в 1 метр друг от друга в глубину 2 метра и ширину 100мм. Первые 1.5 метра заполняются крупным боем бетона, внутри этой подсыпки будет примерно 5 кубометра пространства которое будет быстрым дневным буфером для воды, с помощью него 100мм трубы с дырками(дренажные) обмотанные геотекстилем. В течении дня без затыков загонят теплоту в грунт, с вечера и до утра эта вода будет уходить в бока на глубину. Но нужно делать доп расчет на всасываемость и подгонять дренаж и буфер под локацию. Глина сосет плохо, песок хорошо. Далее все это накрываем щебнем, а фундамент не утепляем. Таким образом, за летний сезон мы загоним более 30 МЕГАВАТТ ТЕПЛОВОЙ ЭНЕРГИИ ПОД ДОМ!, 300-400 кубов грунта будет градусов 40! теплота с глубины полуметра будет идти медленно и расширяться во все стороны, потери будут 50%, но по итогу мы получаем теплый пол на халяву на всю зиму. опционально можно защитить территорию вокруг дома от теплопотерь и дождей с помощью утепленной отмостки с гидроизоляцией.
ЧТО ДУМАЕТЕ? Я ЕБНУЛСЯ?
Аноним 24/02/19 Вск 22:55:10 461112454
>>461111


Кстати так можно строить теплицы, можно собирать урожай картохи в январе например в Мурманске, только лампы освящения от ветрячка запитать.
Аноним 24/02/19 Вск 22:57:50 461113455
>>461112


Хотя зачем, можно прямо на земле коллектора сделать для теплиц.
Аноним 24/02/19 Вск 23:36:18 461115456
>>461111
Это не имеет смысла, так как у тебя еще куча земли непаханной в теплых регионах.
Аноним 24/02/19 Вск 23:41:05 461116457
>>461115


Ещё как имеет, люди в России живут на северах.


Такой метод потенциально кратно сокращает издержки. Повышает благосостояние и улучшает климат.
Аноним 24/02/19 Вск 23:57:44 461117458
>>461116
>Ещё как имеет, люди в России живут на северах.
Дешевле вырастить в теплом регионе и привезти за один день.
>Такой метод потенциально кратно сокращает издержки. Повышает благосостояние и улучшает климат.
лол
Аноним 25/02/19 Пнд 00:00:10 461118459
>>461117

>Дешевле вырастить в теплом регионе и привезти за один день.


Цены в Мурманске на огурцы в январе- ну так себе.
Аноним 25/02/19 Пнд 01:26:53 461126460
вкатываюсь со стат. физикой
Вывести формулу для <(N-<N>)^k>, где k -целое, через параметры бол. кан. распред.
Вычислить флуктуации p^2, s^2 и ps
может кто знает как ето решается. или хотя бы посоветуйте методичку для решения такого рода задач, так как я даже не понимаю как подступиться
Аноним 25/02/19 Пнд 02:34:02 461129461
>>461118
>Цены в Мурманске на огурцы в январе- ну так себе.
Дешевле чем растить на этой штуке.
Аноним 25/02/19 Пнд 02:36:29 461130462
>>461126
Ну тебе надо теорвер понять это во-первых. А во-вторых, открой учебник Борщевского. Если надо моменты расчитать, найди функцию производящих моментов от большого канонического распр.
Аноним 25/02/19 Пнд 16:44:06 461151463
15353235451650.gif (1350Кб, 387x263)
387x263
Правда ли, что сильные эмоциональные реакции (на всякую хуйню, которая не критична) свойственны лишь людям с низкими когнитивными способностями? Я знаю, что есть полные овощи без эмоций, но люди, которые испытывают испанский стыд, сильно краснеют и смущаются при соц. взаимодействиях, не имеют достаточно развитого мозга, который бы просто фактом своего наличия гасил все переживания или не уделял внимания чему-либо, что не пересекаeтся с личными интересами, так?
Аноним 25/02/19 Пнд 17:47:14 461152464
Анон. Боюсь пробовать. С собеседованиями более-менее справился, потому что меня приглашают на стажировки (и на дно-работы, и были шансы практически по специальности, на неплохую для моего города должность устроиться через друга), но я прихожу с собеседования домой и меня охватывает такая паника, что я в итоге отказываюсь. Боюсь, что не справлюсь, при чем не важно, расставлять банки по полкам в соседнем универмаге или проектировать космические ракеты. Сейчас думаю попробовать вкатиться в копирайтинг, чтобы хоть какую-то денежку иметь, но даже на сайте зарегистрироваться и пройти тестовое задание СТРАШНО. Посоветуй, что с этим сделать можно, пожалуйста?
Аноним 25/02/19 Пнд 17:48:46 461153465
>>461152
Так, я промахнулся доской, извините. Но если вдруг кто-нибудь ответит, буду благодарен
Аноним 25/02/19 Пнд 18:53:27 461154466
Вселенная бесконечна и безгранична?
Аноним 25/02/19 Пнд 20:36:34 461161467
>>458700 (OP)
Относительно недавно научились превращать твёрдые отходы во всякие полезности.
Когда же научатся преобразовывать жидкую и газообразную фракции из токсичного говна в полезную хуйню?
Аноним 26/02/19 Втр 00:15:36 461171468
>>461154
>бесконечна
Нет
>безгранична
Ей не с чем граничить, так что хз
Аноним 26/02/19 Втр 02:03:35 461172469
После того как я потрахаю себя дилдой около 10 минут, мне становится спокойно, будто я получил оргазм, но я его не получал. Но оргазм я не испытывал, просто долбил себя немного. Но состояние такое будто я 10 раз кончил, но нет чувства опустошенности как при обычном оргазме хуем. Может ли быть какая то связь между этим?
Аноним 26/02/19 Втр 11:24:17 461181470
800px-Ssc2007-1[...].jpg (184Кб, 800x695)
800x695
>>461154
>бесконечна
Мы не знаем
>безгранична
Мы не знаем
Аноним 26/02/19 Втр 11:25:51 461182471
15408857395033.jpg (98Кб, 1080x1080)
1080x1080
>>461152
Да прост делай без задней мысли, потом привыкнешь.
Надо выходить из зоны комфорта.
Аноним 26/02/19 Втр 12:06:55 461184472
Такой вопрос:

есть тут кто с технических образованием? (математика, физика, химия, биология) (наверняка все присутствующие)

и чтоб разбирался в философии 20го и 21го веков? Расскажите, что читали изучали, и как оно вам?
Аноним 26/02/19 Втр 12:26:37 461185473
я уточню
что мне не интересно мнение про типичных философов, которых здесь вспоминают (типа Куна, Поппера и Фейерабенда, тут всё понятно)
интересно (нетривиальное) мнение (в смысле что вы серьёзно читали их и нашли что-то всё-таки полезное и интересное) про континентальных, аналитических философов, и тех, что творя сегодня (типа спекуляционистов, акселерационистов и прочих)

(мнение социологов тоже интересно, хотя их тут наверно нет)
Аноним 26/02/19 Втр 12:46:56 461188474
image.png (77Кб, 229x306)
229x306
>>461185
>>461184
Нахуй пошел со своими философами отсюда
Аноним 26/02/19 Втр 12:59:27 461189475
>>461184
>и чтоб разбирался в философии 20го и 21го веков? Расскажите, что читали изучали, и как оно вам?
Под долгу службы читал людей имеющих отношения к философии науки. Более отборного бреда, состоящего из очевидных утверждений и ужасающего дилетанства нигде не видел и не читал. После того как сдал этот бред, больше к философии науки не притрагивался и не желаю. Знаете ли тыкать говно такое себе удовольствие.
Также открыл для себя такого человека как Отто Вейнингер, не знаю можно ли считать его философом, но его произведения вполне себе доставили.
Аноним 26/02/19 Втр 13:13:15 461190476
Я правильно понимаю, что филосовствовать в науке можно только в тех областях, о которых мы еще все до конца не знаем или не узнаем совсем?
Аноним 26/02/19 Втр 13:26:26 461191477
Безымянный.png (79Кб, 1284x743)
1284x743
План 100% автономии

- Подбирается участок, ровный, с песком, на высоте, с глубокими грунтовыми водами

- черная металлическая односкатная крыша на юг 120м2
- на неё равномерно сбрасывается вода с помощью шлагна
- дополнительно накрывается тепличной пленкой для повышения КПД и снижения испарения
- вода выше 35 градусов всё лето сбрасывается в огромное дренажное поле 12 на 12 метров, около 100мвт энергии
- грунт прогревается на 5 метров вниз, почти тысяча кубометров грунта прогретого выше 36 градусов, огромная инерция, земля греет фундамент-плиту без утепления всю зиму
- далее вокруг дома, для возможности обслуживания пускаем шланг на глубине 30см и накрываем утеплением сверху
- этот шланг в 100 метров загонит 1.5квт/ч 30 градусов теплоты в 200втный самодельный тепловой насос, который с коэффициентом 1:7-8 будет загонять воду в 70градусов 1.5квт/ч градусов в бойлер
- из бойлера мы так же кидаем теплообменник на огромный низкотемпературный двигатель стирлинга диаметров 2-3 метра, он будет цепью опущен в сухой колодец с постоянной низкой температурой
- с кпд в 30-40% он выдаст 500вт, то есть 300-400вт/ч энергии в плюс, эти 300вт будут 24/7 загоняться в литий-ионный аккумулятор в 5-10Квт/ч чего полностью хватит.

Итого:
полное отопление дома
горячая вода
электричество из стабильного источника


Критикуйте, напомню, что чем больше объем тепло аккумулятора - тем дольше он остывает, тут будет более 1000 кубов горячей земли и утепление сверху, этот монстр не остынет до весны.
Аноним 26/02/19 Втр 14:06:14 461193478
>>461191


Да, скважинка будет качать от солнечной панели летом*
Аноним 26/02/19 Втр 14:51:59 461194479
Аноним 26/02/19 Втр 15:03:08 461195480
>>461184
>как оно вам?
"Многих работающих математиков смущает вопрос, чем же являются доказательства, если они не могут доказывать. С одной стороны, они знают из опыта, что доказательства могут быть ошибочными, а с другой, — по своему догматистскому углублению в доктрину они знают, что подлинные доказательства должны быть безошибочными. Математики-прикладники обычно решают эту дилемму застенчивой, но крепкой верой, что доказательства чистых математиков являются «полными» и что они действительно доказывают. Чистые математики, однако, знают лучше — они уважают только «полные доказательства», которые даются логиками. Если же их спросить, какова же польза или функция их «неполных доказательств», то они большей частью теряются."
Аноним 26/02/19 Втр 16:43:06 461198481
>>461184
Короче, есть два типа учёных:
1) Те, кто считают, что методология - это абсолют, что им даны все истины, и что сомневаться в этих истинах - грешно;
2) Те, кто не считают зазорным сомневаться в самых фундаментальных истинах.
Для примера: Фейнман говорил, что философия бесполезна; а Ли Смолин (например, "Time Reborn") - что то, что делают физики, увязло в идеализированных математических абстракциях и к реальности отношения может иметь не больше, чем идеально круглые траектории планет.
Аноним 26/02/19 Втр 19:28:22 461204482
>>461198
Первый так себе пример. Можно запросто сомневаться хоть в том что вода мокрая и при этом не надрачивать на нелепые фейловысеры. Более того ни один фейловысер никогда не поможет продвинуться в понимании физической теории. В этом собственно и смысл утверждений физиков о ненужности философии. Разве что порадоваться за фейломанек как они могли сомневаться как не всякому дано, наверное.
Обычно выходит так что конечно же каждый себя вносит в категорию 2 (ведь он не тупое быдло) но на деле выходит 1. Достаточно чуть-чуть копнуть вглубь и включается защитная реакция - мол это все просто не нужно. Если продолжать давить - было проверенно мной на практике многократно - может только получиться безразмерный срач и куча порванных пердаков. Наглядный пример чтобы далеко не ходить - тред про наблюдателя.


Аноним 26/02/19 Втр 19:33:29 461205483
Посоветуйте учебник по математике для инженера, сдававшего ее 10 лет назад. Хочется вспомнить пройденное и по-нормальному ее изучить, а не из принципа "сдал-забыл". Можно еще отдельно посоветовать учебник по статистике и теории вероятности.
Аноним 26/02/19 Втр 19:45:27 461207484
Почему одноклеточных не едят ну кроме дрожжей каких-нибудь, но они типа приправы скорей? Потому что они срут под себя и от говна не избавиться?
Аноним 26/02/19 Втр 19:54:48 461209485
Lee Smolin.jpg (171Кб, 755x709)
755x709
Lee Smolin2.jpg (62Кб, 661x289)
661x289
>>461204
>ни один фейловысер никогда не поможет продвинуться в понимании физической теории
Зато поможет в ней усомниться и отфильтровать хуиту. Потому что клоуны её безосновательно переусложняют всякими теориями струн да пытаются применять для ответа на принципиально неотвечаемые метафизические вопросы - чего они могли бы избежать, если бы хоть немного разбирались в философии.

https://en.wikipedia.org/wiki/David_Albert#Feud_with_Lawrence_Krauss
Аноним 26/02/19 Втр 20:39:38 461214486
>>461209
Какая то битва двух петухов (даже трех). В которой местная фейломанька наверняка снова поняла все по-своему - т.е. ровно наоборот чем оно есть на самом деле. Даже и разбираться не буду.
Аноним 26/02/19 Втр 20:43:56 461215487
>>461214
>Какая то битва двух петухов
И все с Ph.D. по физике. Вот ведь незадача.

>Даже и разбираться не буду
Ясен пень, не твой уровень, быдло. Тебе лишь бы "фейломанек" на двачах детектить.
Аноним 26/02/19 Втр 21:39:48 461221488
>>458700 (OP)
Почему между многими видами млекопитающих возможно скрещивание с созданием потомства (конь+ослица/коза+овца/тигр+лев), но только не между человеком и человекообразными обезьянами?
inb4: на самом деле тоже возможно, но власти скрыва
Аноним 26/02/19 Втр 22:33:04 461228489
Ричард Фейнман [...].mp4 (9946Кб, 424x240, 00:10:05)
424x240
>>461198
>Те, кто считают, что методология - это абсолют, что им даны все истины, и что сомневаться в этих истинах - грешно;
>Приводит пример Фейнмана
Ебать ты чмо. Сам научный метод отрицает сам себя, тащемто, скорее постулируя что "истины нет".
Аноним 26/02/19 Втр 22:34:19 461229490
>>461204
>Наглядный пример чтобы далеко не ходить - тред про наблюдателя.
Там просто 5% понимают матан и кванты. Остальные 95% это просто уебки, насмотревшиеся научпопа.
Аноним 26/02/19 Втр 23:16:35 461240491
>>461228
Фейнман сомневается в теориях, но ему не приходит в голову сомневаться самой в методологии, при помощи которой он согласует свои эксперименты.

Так-то, например, ещё Ньютоновская механика устраняет время, потому что черчение времени на графике превращает его в пространство (т.н. spatialization of time), т.е. описывает некое упрощённое пространство, изначально лишённое времени. И в этом контексте, теория относительности Эйнштейна, официально устраняющая время, действует также уже изначально в рамках некоего пространства, временем никогда не обладавшим.
Аноним 26/02/19 Втр 23:56:39 461249492
Я вот тут читал о катализаторах и, неожиданно для себя, загуглил новое умное слово -промотор. И тут пришла в голову мысль. А что если допустить, что есть некое вещество, которое увеличивает активность ацетихолинэстеразы так, что ацетилхолин нахуй разлетается в клочья быстрее,чем успевает с чем-то там связаться. Это что ж такое будет, много ли я ъуйни сейчас сказал(Знаю,что много, но интересует примерный процент) и как это будет действовать на человека? Вот так как-то.
Аноним 27/02/19 Срд 00:38:28 461259493
>>461240
>не приходит в голову сомневаться самой в методологии
Ну и с чего ты взял? Вся наука, это в первую очередь развитие методологии, нежели накопление знаний.
Аноним 27/02/19 Срд 01:00:39 461261494
Давайте я перезадам вопрос про философию. Интересно мнение про философов, которых как-то никогда не обсуждают на научных досках и вообще.

Есть такая штука, как наука, которая много чего объясняет и делает это очень клёво и в общем она работает.

Есть философия науки, которая меня в данном вопросе не интересует.

А есть прочая философия, которая не вполне понятно как совмещается с наукой. Ну аналитические философы может ещё куда ни шло, они вроде как типа рассуждают логически, хотя с другой стороны, ни один не согласен с другим, да и читать их как-то уныло, имхо.

Есть континентальные философы, которых обычно принято ругать только. Это немного странно, потому что вроде бы как всё-таки умные гуманитарные люди их читают. Что-то же они там находят?

И есть современные философы, типа
- анти-корреляционистов (или объектно-ориентированные философы)
- акселерационисты (левые (Срничек), правые (Ланд), универсальные (хз кто тут))
- прочие (Реза Негарестани, например)

Или так.
До 20го века была какая-то философия, её вполне себе изучают в универе, и люди как-то с этим смиряются видимо, поэтому вопрос опять же не про Канта и не про Гегеля и даже не про Кьеркегора, хотя тут кажется никого не упоминали. Витгенштейн тоже не интересен в данном вопросе, не могу объяснить почему, но мне просто кажется с ним всё и так понятно и его даже вполне читают и норм чувак, он так-то технарь сам.

Мне интересно ваше мнение, например, если кто читал и изучал, о таких прям конкретно даже философах как:
(хуй с ним с Хайдеггером)
- Симондон (что-то типа философия техники)
- Делёз (объелся слёз)
- Латур (придумал акторно-сетевую теорию, написал милую книгу про Пастера)
- Мейясу (суперпопулярен сегодня, и написал неплохую книжку "После конечности"; он там как-то хитро пользуется логикой, мне интересно ваше мнение на этот счёт)
- Прист (неклассические логики)
- Уайтхед
- Лакан (зачем-то использовал термины из элементарной топологии)
- прочие аналитические философы после Витгенштейна, типа Квайна, Чёрчленда, Крипке (в данном вопросе мне не сильно интересны учёные, которые рассуждают про мозг, типа Деннета, сорян)
- прочие континентальные или смесь континентального-и-аналитического (Джон Когбёрн или Тристан Гарсиа или Реза Негарестани)

вот. короче я не понимаю. популярность у них среди вроде как умных или интеллектуальных или рассуждающих людей есть, но какого-то диалога с наукой или влияния на неё я вообще не вижу.

Ну типа, чё как, чё читали, считаете ли вы это жизненным, что думаете про то, что условно гуманитарии упарываются и вдохновляются больше философией, чем наукой и прочее


[ну я так-то много чего вообще не знаю, типа есть какая-то мета-этика (что это)
наверняка ещё есть какие-то классные современные философские работы про время и пространство и про мб даже квантовое, которые наверно и физикам были бы релевантны, но я не в курсе таких кажется]
Аноним 27/02/19 Срд 01:10:20 461262495
churchland paul[...].jpg (102Кб, 783x487)
783x487
churchland paul[...].jpg (202Кб, 453x765)
453x765
churchland paul[...].jpg (215Кб, 449x767)
449x767
churchland paul[...].jpg (60Кб, 459x343)
459x343
>>461261
>типа Квайна, Чёрчленда, Крипке (в данном вопросе мне не сильно интересны учёные, которые рассуждают про мозг, типа Деннета, сорян)
>Чёрчленда
>мне не сильно интересны учёные, которые рассуждают про мозг, типа Деннета
Ебан штоле?
Аноним 27/02/19 Срд 01:17:29 461263496
>>461261
> Интересно мнение про философов, которых как-то никогда не обсуждают на научных досках
Потому что они не рождают знания.
У тебя есть своя доска, иди туда.
Аноним 27/02/19 Срд 01:40:12 461264497
Аноним 27/02/19 Срд 01:42:11 461265498
>>461263
> Потому что они не рождают знания.
можешь раскрыть подробнее тезис?

ну то есть, мне непонятна эта дихотомия науки и философии, кажется она не про порождение знания всё же (ну иначе кажется философия нахуй не нужна, я не против такой точки зрения, просто, скажем так, будем считать это слишком радикальным решением и наивным)
Аноним 27/02/19 Срд 01:52:57 461266499
>>461265
>можешь раскрыть подробнее тезис?
Философия не привязана ни к чему. На любой тезис там есть антитезис, который просто нечем опровергать. Это просто бесполезно.
Аноним 27/02/19 Срд 02:45:27 461269500
Чуваки, чуваки, слушайте. А может ли в теории существовать крысопаук? Типа выглядит, как паук, но ведёт себя как крыса? Зерно там всякое грызет, и прочее
Аноним 27/02/19 Срд 04:20:44 461278501
>>461221
Потому что люди и обезьяны не настолько близкородственны, как виды в твоих примерах. Вот с неандертальцами сапиенсы очень даже скрещивались.
Аноним 27/02/19 Срд 07:47:30 461279502
>>461266
>Философия не привязана ни к чему. На любой тезис там есть антитезис, который просто нечем опровергать

"So what is a philosopher to do, if not troll his mind for conceptual truths? The Quinean answer is this: many things, including synthesizing across various subfields and theorizing while immersed in and constrained by available facts." (c) Патриция Чёрчленд
Аноним 27/02/19 Срд 09:51:58 461283503
>>461261
>но какого-то диалога с наукой или влияния на неё я вообще не вижу.
Потому что пиздеть не мешки ворочать.
Аноним 27/02/19 Срд 09:56:45 461284504
>>461269
Про крысопауков не знаю, но видел какую-то серьезную документалку про крысокана. Он там в каком-то заброшеном мотеле в США поселился и на него охотились.
Аноним 27/02/19 Срд 10:12:11 461285505
>>461283
>пиздеть не мешки ворочать
https://en.wikipedia.org/wiki/Experimental_philosophy

>>461261
>есть какие-то классные современные философские работы про время и пространство и про мб даже квантовое, которые наверно и физикам были бы релевантны, но я не в курсе таких кажется
Ли Смолин (Ph.D. по физике) https://en.wikipedia.org/wiki/Time_Reborn
Дэвид Альберт (Ph.D. по физике) https://en.wikipedia.org/wiki/David_Albert
Философия физики, так-то.

>Есть континентальные философы, которых обычно принято ругать только. Это немного странно, потому что вроде бы как всё-таки умные гуманитарные люди их читают. Что-то же они там находят?
Континентальные в плане своей методологии отталкиваются от феноменологии и прочих априорно-метафизических, трансцендентальных и антропоцентрических допущений. Отсюда и все проблемы.
Аноним 27/02/19 Срд 10:18:35 461289506
>>461285
>https://en.wikipedia.org/wiki/Experimental_philosophy
Все еще - пиздеть не мешки ворочать.
>Философия физики, так-то.
Только читает ли эти опусы хоть кто-то из физиков? Сомненваюсь. Обычно подобные "работы" попытки срубить бабла с гоев.
Аноним 27/02/19 Срд 11:16:09 461312507
>>461285
ли смолин наверно и философ, но в первую очередь он физик,
типа не совсем то, что я имел в виду, но спасибо
Аноним 27/02/19 Срд 12:46:28 461347508
даже так
ли смолин - философ в том же смысле, что эйнштейн философ, фейнман философ.

но

это не то, что я ищу:

во-первых, ли смолин конкретно выступает за петлевую квантовую гравитацию, по-крайней мере в физических работах, то есть он прям конкретный физик, даже с какими-то уравнениями, пеной вот это всё, хоть и теоретический

во-вторых, это всё физики философы, которые вообще никак не ссылаются и не полемизируют с остальными философами, которые философы, которые котируются у гуманитариев (делёз и прочие)

и да, подозреваю, что гуманитарии не читают смолина.

Я знаю два примера математиков, которые читали условного Делёза:
- Рома Михайлов - тут понятно, открываешь изнанку крысы и всё видно
- Фернандо Заламеа - Synthetic Philosophy of Contemporary Mathematics - трудноватое чтиво, он типа продвигает идею, что пучки - это круто, надо их в философию засунуть. Ну не думаю, что его вообще в мире много кто читал.

(сорян ещё за рассуждения уровня "не читал но осуждаю", но подозреваю, что Дэвид Альберт тоже оторван от диалога с остальной философией)
(я не говорю, что это плохо или неправильно,
но меня смущает такое положение дел) (когда у одной интеллектуальной тусовки есть свой круг тем, у другой свой, и все считают, что они правы, а остальных они скажем так игнорят)
Аноним 27/02/19 Срд 14:51:04 461415509
>>461347
> которые вообще никак не ссылаются и не полемизируют с остальными философами
Если хочется языком почесать, можно и с корешами в баре посидеть, попить пива. Пользы будет больше.
>когда у одной интеллектуальной тусовки есть свой круг тем, у другой свой, и все считают, что они правы, а остальных они скажем так игнорят
Cитуация несколько иная. Есть условные физики, которые решают реальные проблемы/задачи и.т.д Т.е пишут формулы, проводят качественный и количественный анализ и.т.д И есть "философы", которые кроме как пиздеть ничего не умеют. И это абсолютно нормально, что у каждый своя тусовка, ибо вторым нечего делать у первых, а первым, как я уже писал, проще с корешами пива попить.
Аноним 27/02/19 Срд 15:33:10 461429510
>>461347
>философ в том же смысле, что эйнштейн философ
https://plato.stanford.edu/entries/einstein-philscience/

>условного Делёза
Условный Делёз - это постмодернизм (а конкретный - ещё и априорная метафизика с "ризомой"). А постмодернизм - это такая своеобразная философия языка. Так что, возмущаться что физики не котируют Делёза, это как возмущаться, что Делёз не упоминает ворох фамилий из какой-нибудь философии математики.
Во-вторых, постмодернизм на ура срабатывает только там, где основным научным методом является интерпретация (типа социологии, археологии, социальной антропологии) - но в точных науках слишком узкое пространство для интерпретаций, так что финты ушами типа "давайте представим, что квадрат - это круг" без поломки работоспособности системы там не канают.
Прибавить к этому, что на весь постмодернизм там только 2 годные фамилии (Фуко и Деррида), но и те пишут, как нечитаебельные ебанавты. Деррида, например, тезисно совпадает с Куайном почти до полного буквализма, но писал он при этом так нечитабельно, что Куайн даже какое-то письмо подписывал, в котором громил Дерриду, как лже-философа.
Аноним 27/02/19 Срд 16:53:56 461437511
Как определяется электроемкость металла? Если взять цельный куб металла и цельный куб золота, где можно будет сохранить больше энергии, и почему?
Аноним 27/02/19 Срд 18:33:08 461450512
>>461429
о, ты читал Делёза, надо думать
как тебе?
ну типа понятно, что Делёз - не про математику (он сам признавался, что не знает её), и вообще не про науки.
Но типа же он что-то вещает, какое-то мировоззрение. Детерриториализацию вот это всё. Ризома, да.
Создаёт концепты.
Как тебе его концепты? Это ощущается полезным?
Аноним 27/02/19 Срд 21:51:17 461473513
кто все еще занимается наукой хоть как-то, но вынужден работать инженером/программистом, расскажите как у вас получается?
Аноним 27/02/19 Срд 22:25:16 461475514
>>461473
Тут 95% борды занимается на мейлаче наукой и гоняет верунов, а ирл моет посуду в макдаках.
Аноним 28/02/19 Чтв 01:58:17 461481515
Ладно, ещё вопрос,

каково ваше мнение по психоаналитике? Прибегали ли вы к помощи психоаналитика, помог ли он вам, считаете ли вы это нормальным или антинаучной хуйнёй, может читали Лакана, лол?
Аноним 28/02/19 Чтв 07:42:25 461483516
Аноним 28/02/19 Чтв 09:51:47 461487517
>>461483
это же разные вещи
Аноним 28/02/19 Чтв 12:44:35 461506518
>>461487
>human capacity to explain and predict the behavior and mental state of other people
>Ид, Танатос, Другой, эдипов комплекс, архетипы
Нет, это то самое.
Аноним 28/02/19 Чтв 15:37:05 461513519
>>461475
работал с phd из США. работа не научная. разговора ради, обсуждали как ставили эксперименты в выходные. им лет по 50. но два дня в неделю, этого же мало?

читал про академическую карьеру, но мне уже поздно. 35 лвл

год работать, год не работать. но время идет и мне кажется, чем дальше, тем труднее будет искать работу.

наверное я что-то простое не понимаю, раз не нахожу обсуждений этой темы.
Аноним 28/02/19 Чтв 20:01:57 461526520
>>461151
Нет, ты придумал глупость.
Смотри, внешнее проявление эмоций - это следствие социальных навыков, без социальных навыков ты можешь бинарно а)орать в обосранном подгузнике б) не орать в чистом.
Кто лучше всего прокачивает социальные навыки? Конечно же экстраверты. Как связаны экстраверсия и интроверсия с уровнем IQ? Никак.
Аноним 28/02/19 Чтв 20:24:09 461528521
Во сколько раз матан теории струн сложнее стандартной модели? Или они весьма сопоставимы?
Аноним 28/02/19 Чтв 21:00:54 461530522
>>461528
Работу пишут в кпт, суперструны считал по учебнику только на уровне ознокомления. Сложность мат. апарата примерно на таком же уровне.
Настройки X
Ответить в тред X
15000 [S]
Макс объем: 40Mб, макс кол-во файлов: 4
Кликни/брось файл/ctrl-v
Стикеры X
Избранное / Топ тредов